Всероссийская олимпиада школьников по литературе 2018 2018 ответы 5 класс: Задания, критерии , ответы школьного тура олимпиады по литературе 5 класс

Содержание

Задания, критерии , ответы школьного тура олимпиады по литературе 5 класс

Всероссийская олимпиада школьников по литературе

Школьный этап 2017-2018 учебный год

5 класс

I . Знание текстов художественных произведений.

Задание 1. Определите, к какому жанру фольклора принадлежит каждый из текстов:

А. Привела ведьма Алёнушку на реку. Кинулась на неё, привязала Алёнушке на шею камень и бросила её в воду.А сама оборотилась Алёнушкой, нарядилась в её платье и пришла в её хоромы.

Ответ:________ ___

Б. Шла машина темным лесом.

За каким-то интересом.

Инте-инте-интерес,

Выходи на букву «ЭС».

Ответ:________ ______

В. На свете нет ее сильнее,

На свете нет ее буйнее.

В руках ее не удержать —

И на коне не обогнать.

Ответ:________ _______

Г.

Дерево держится корнями, а человек друзьями.

Ответ:________ ______

Задание 2. Подберите к данным текстам подходящие «крылатые слова», взятые из басен.

1) Задиристый малыш повздорил с гораздо более сильным и рослым противником. Окружающие смеются:……

2) Перед баскетбольным матчем между школьными командами нападающий одной команды хвастался в раздевалке: «Да мы их обыграем обязательно!» Его команда проиграла. Огорчённые товарищи бурчали:…..

3) Вместо подготовки к контрольной работе Катя пошла на дискотеку и веселилась там весь вечер. Назавтра на контрольной по математике, обращаясь за помощью к соседям, она услышала:….

4) Мальчик, не зная урока, исписал всю доску, пытаясь решить задачу, наконец, учитель написал на доске правильное решение и со словами:…….. отдал ему дневник с двойкой.

а) «Хвалилась синица море сжечь»

б) «А ларчик просто открывался»

в) «Ай, Моська! Знать она сильна, что лает на слона!»

г) «Ты всё пела? Это дело: так поди же, попляши!»

Ответ запишите парами соответствующих цифр и букв (например: 1-г).

Ответ: ___________________________________________________________

__________________________________________________________________

II. Историко-литературные задания.

Задание 3.Определите писателя по биографическим фактам.

А) Воспитывала его бабушка, учился в Московском университете, поступил в школу гвардейских подпрапорщиков и кавалерийских юнкеров, за одно свое произведение был сослан на Кавказ, погиб на дуэли.

Ответ: ________________________

Б) Он вошёл в историю литературы своей стихотворной сказкой, хотя остальные его произведения забыты. Этой сказкой о необыкновенном коне восхищался Пушкин, зачитываются ею и наши современники.

Ответ: ________________________

В). Родился в Москве. Семья жила бедно. Мать рано его приучила к чтению. Рано умер отец. За время своей жизни был журналистом, издателем, автором многих прозаических и драматических произведений. Но в большей степени известен как баснописец.

Ответ:________________________

Задание 4. В квадрате зашифрованы фамилии русских писателей. Читаться они могут слева направо, справа налево, по горизонтали или вертикали, могут «ломаться» под прямым углом. Найдите эти фамилии.

Ж

у

к

Б

а

ж

н

с

А

о

в

с

о

и

т

Б

а

р

к

в

к

а

х

е

а

и

у

ш

ф

о

Ч

т

й

П

Н

ь

в

й

ы

н

в

о

е

в

и

к

с

о

с

Ответ:______________________________________________________________________________________________________________________________________________________________________________________________________

III. Знание теории литературы.

Задание 5: Какое средство художественной выразительности использовано в отрывке из стихотворения М.Лермонтова :

А. Ночевала тучка золотая

На груди утеса-великана,

Утром в путь она умчалась рано,

По лазури весело играя.

Б.  Какой троп использован в отрывке из былины, в котором изображается сила богатырского коня: 

Первый скок нашли за три версты,

Другой скок нашли за двенадцать вёрст,

Третий скок не могли найти.

В.  Какое средство художественной выразительности использовано в отрывке произведения П.П.Ершова «Конёк – горбунок».

Кабылица та была

Вся, как зимний лес, бела,

Грива в землю, золотая,

В мелки кольца завитая…»

Ответ:______________________________________________________________

Задание 6. Соотнесите термин и его определение.

Ответ запишите парами соответствующих цифр (например: 1-4).

Термины

Определения

1.Юмор

1.Жанр фольклора, краткое изречение с назидательным содержанием.

2.Басня

2. Краткое образное выражение, незаконченное высказывание

3.Пословица

3.Короткий рассказ, чаще всего стихотворный, иносказательного содержания.

4. Поговорка

4. Описание природы в литературном произведении

5. Пейзаж

5.Вид комического, весёлый, безобидный, смех.

Ответ: ___________________________________________________________
__________________________________________________________________

IV. Литература и другие виды искусства.

Задание 7. Великолепные картины В.М.Васнецова на сказочные сюжеты могут без слов рассказать о волшебстве и коварстве, о любви и невежестве и о вечном торжестве добра. Перечислите картины из цикла «Поэма семи сказок»

Ответ: ___________________________________________________________
___________________________________________________________________________________________________________________________________
__________________________________________________________________

V. Творческое задание.

Задание 8. В стихотворении перепутаны строки.

а) Восстановите  стихотворение:  перепишите  строки  в  правильном  порядке

Подсказка 1: знаки препинания в концах строк.

Подсказка 2: рифмовка в стихотворении парная.

б) Обозначьте жанр стихотворения, которое Вы восстановили. Как называется этот жанр?

Ответ: _________________________________________________________

в) О каком предмете идёт речь в стихотворении?

Ответ : _________________________________________________________

Официальный сайт МБОУ «Школа №8» г.Ачинска

2017-2018 учебный год

Всероссийская олимпиада школьников

Приказ №1488 от 17.12.2015 «О внесении изменений в Порядок проведения всероссийской олимпиады школьников» читать>>

Приказ №134 от 23.04.2008 «Об утверждении перечня общеобразовательных предметов, по которым проводится всероссийская олимпиада школьников»

читать>>

Приказ от 18 ноября 2013 г. № 1252 «Об утверждении порядка проведения всероссийской олимпиады школьников» читать>>

Приказ от 12 сентября 2017 г. № 351 «О проведении школьного этапа всероссийской олимпиады школьников» читать>>

Школьный этап (5-11 классы) Приказ №105 от 14.09.2017 г. «О проведении школьного этапа всероссийской олимпиады школьников» читать>> Заявления об участии в школьном этапе читать>> Победители и призеры школьного этапа читать>>  Муниципальный этап (7-11 классы) Заявления об участии в муниципальном этапе читать>> Региональный этап (9-11 классы)

2016-2017 учебный год

Всероссийская олимпиада школьников

Школьный этап (5-11 классы) Приказ №106 от 13.09.2016 г. «О проведении школьного этапа всероссийской олимпиады школьников»
читать>>
График проведения школьного этапа читать>> Заявления об участии в школьном этапе читать>> Победители и призеры школьного этапа читать>> Работы победителей и призеров школьного этапа смотреть>> Отчет о проведении школьного этапа читать>> Победители и призеры, которые проходят на муниципальный этап ВсОШ-16 читать>>

 

2015-2016 учебный год

Предметные олимпиады в начальных классах (2-4 классы)

Всероссийская олимпиада школьников (5-11 классы)

Школьный этап (5-11 классы) Приказ №60/2 от 04. 09.2015 г. «О проведении школьного этапа всероссийской олимпиады школьников» читать>> График проведения школьного этапа читать>>

Протоколы школьного этапа: (*информация размещена с письменного согласия родителей, законных представителей учащихся

)

Муниципальный этап (7-11 классы)

2014-2015 учебный год

Школьный этап (5-11 классы) Муниципальный этап (7-11 классы)

Результаты муниципального этапа всероссийской олимпиады школьников — 2014

 

2013-2014 учебный год

Предметные олимпиады в начальных классах (2-4 классы)

Школьный этап

Всероссийская олимпиада школьников (5-11 классы)

Региональный этап (9-11 классы) Приказ Министерства образования и науки РФ № 1304 от 04.12.2013 г. Об установлении сроков проведения регионального этапа всероссийской олимпиады школьников по общеобразовательным предметам в 2013-2014 уч.
г. читать>> Муниципальный этап (7-11 классы) Результаты муниципального этапа  читать>> Школьный этап (5-11 классы) Приказ №177 от 01.10.2013 «О проведении школьного этапа всероссийской олимпиады школьников» читать>> Cписки победителей школьного этапа олимпиады  читать>>

 

 

ВСЕРОССИЙСКАЯ ОЛИМПИАДА ШКОЛЬНИКОВ ПО ЛИТЕРАТУРЕ уч. г. ШКОЛЬНЫЙ ЭТАП. 5 КЛАСС Задания, ответы и критерии оценивания

Критерии оценивания и ответы к заданию класс Ответы: Цех поэтов Гилея Обэриуты Серапионовы братья 2, 4, 9 5, 8, 11, 13 3, 7, 10 1, 6, 12

Критерии оценивания и ответы к заданию 1. 11 класс 1.1. Ответы: Цех поэтов Гилея Обэриуты Серапионовы братья 2, 4, 9 5, 8, 11, 13 3, 7, 10 1, 6, 12 За каждый полностью верно записанный столбец по 1 баллу.

Подробнее

Как волк своё получил

Как волк своё получил дна’жды но’чью лиса’ пошла’ в ау’л 1 за ку’рицей. Она’ пошла’ туда’ потому’, что о’чень хоте’ла есть. В ау’ле лиса’ укра’ла* са’мую большу’ю ку’рицу и бы’стро-бы’стро побежа’ла в

Подробнее

Отметьте верный вариант ответа. Время выполнения работы — 90 минут. 1 D, F, E, D, C, A, B 2 D, F, A, C, E, G, B 3 D, F, A, G, B, E, C

ФАМИЛИЯ КЛАСС — ИМЯ Отметьте верный вариант ответа. Время выполнения работы — 90 минут. 1. Определи правильную последовательность событий в русской народной сказке «Лиса и волк». A. B. C. D. E. F. G. 1

Подробнее

Статья «Учимся писать сочинение-сказку»

Статья «Учимся писать сочинение-сказку» Мы живем в мире сбывшихся сказок Сказки не только не умирают они рождаются непрерывно. Рождаются они и сейчас К.Г. Паустовский «Рождение сказки» Цели проведения

Подробнее

Проверяемый планируемый результат

Спецификация проверочной работы 3 по разделам: «Мы — друзья», «Здравствуй, матушка зима», «Чудеса случаются». Предмет: Литературное чтение Программа: Перспектива Класс:2 Цели: проверить знания обучающихся

Подробнее

первой книги обращением

Для читателя Мой добрый, внимательный друг! Перед тобой новый учебник по литературному чтению для 4 класса. В отличие от первых учебных книг почти все произведения здесь ты будешь читать самостоятельно.

Подробнее

СХЕМА ОЦЕНИВАНИЯ ТЕСТА

СХЕМА ОЦЕНИВАНИЯ ТЕСТА Реальный профиль Задание А (40 баллов) Nr Задание Вариант ответа Критерии оценивания Общее количество баллов 1. Определите две основные темы данного фрагмента. Ответ аргументируйте.

Подробнее

УДК 373:82 ББК 83.3я71 К30

УДК 373:82 ББК 83.3я71 К30 К30 Кац, Э. Э. Литературное чтение : тесты и самостоятельные работы для текущего контроля. 4 класс : к учебнику Э. Э. Кац «Литературное чтение» / Э. Э. Кац, Н. А. Миронова. 2-е

Подробнее

Аполлон Николаевич Майков

Литература, 8 класс, 2015-2016 Всероссийская олимпиада школьников по литературе II (муниципальный) этап 2015 2016 учебный год 8 класс Максимальное количество 50 баллов. Задание 1. Выступите в роли составителя

Подробнее

Надежда Щербакова. Ральф и Фалабелла

Надежда Щербакова Ральф и Фалабелла Жил на свете кролик. Его звали Ральф. Но это был необычный кролик. Самый большой в мире. Такой большой и неуклюжий, что не мог даже бегать и скакать, как остальные кролики,

Подробнее

Учимся писать сочинения в начальной школе

Учимся писать сочинения в начальной школе Калинина Ольга Борисовна, заслуженный учитель РФ, учитель методист, автор учебной литературы УМК «Планета знаний» Примерная основная общеобразовательная программа

Подробнее

Тестовые задания Баллы

Прочитайте внимательно фрагмент из поэмы Н. В. Гоголя «Мертвые души» (Том первый, глава одиннадцатая) и выполните задания…. жили в одном отдаленном уголке России два обитателя. Один был отец семейства,

Подробнее

8-й класс. Дорогие участники олимпиады!

Внесите, пожалуйста, ШИФР с регистрационной карты! Муниципальный тур Всероссийской олимпиады по литературе 2016-2017 уч. год 8-й класс Дорогие участники олимпиады! Предлагаем вам выполнить задания, которые

Подробнее

Всероссийская олимпиада школьников 2018-2019 — ДДТ Петроградского района

2018-2019 учебный год

ВСЕРОССИЙСКАЯ ОЛИМПИАДА ШКОЛЬНИКОВ


Распоряжение Комитета по образованию №3512-р от 13.12.2018 «О проведении регионального этапа всероссийской олимпиады школьников в Санкт-Петербурге в 2018/2019 учебном году».

Проходные баллы для участия в региональном этапе всероссийской олимпиады школьников в 2018-2019 учебном году.

Списки приглашенных на региональный этап всероссийской олимпиады школьников в 2018/2019 учебном году:

  • астрономия
  • математика
  • немецкий язык
  • французский язык
  • технология
  • информатика
  • испанский язык
  • экономика
  • история
  • русский язык
  • химия
  • ОБЖ
  • экология
  • биология
  • география
  • китайский язык
  • итальянский язык
  • литература
  • физика
  • право
  • обществознание
  • физкультура
  • МКХ
  • английский язык

Списки приглашенных на городской этап региональных олимпиад школьников в 2018/2019 учебном году:

  • экономика
  • Олимпиада по физике им. Максвелла

Распоряжение администрации Петроградского района Санкт-Петербурга №3603-р от 02.11.2018 «Об организации и проведении районного этапа всероссийской олимпиады школьников в Петроградском районе Санкт-Петербурга в 2018-2019 учебном году».

Результаты районного этапа всероссийской олимпиады школьников в 2018-2019 учебном году:

13.11.2018 Результаты по истории

14.11.2018 Результаты по немецкому языку

15.11.2018 Результаты по французскому языку

16.11.2018 Результаты по литературе

19.11.2018 Результаты 1 тура по химии

20.11.2018 Результаты по ОБЖ

21.11.2018 Результаты по русскому языку

22.11.2018 Результаты 1 тура по биологии

26.11.2018 Результаты по искусству (МХК)

28.11.2018 Предварительные результаты 7-9 классов по физике

28. 11.2018 Результаты по обществознанию

29.11.2018 Результаты 7-8 классов и списки приглашенных 9-11 классов на 2 тур по биологии

30.11.2018 Результаты по праву

03.12.2018 Результаты 7-8 классов 1 тура по технологии и 5-6 классов олимпиады «Азбука мастерства».

06.12.2018 Результаты по географии

06.12.2018 Результаты по экономике

06.12.2018 Результаты 7-8 классов по технологии и 5-6 классов олимпиады «Азбука мастерства»

10.12.2018 Предварительные результаты по английскому языку

11.12.2018 Результаты по испанскому языку

12.12.2018 12.00 Обновленные результаты по химии

13.12.2018 15.00 Обновленные результаты по информатике

13.12.2018 Результаты по китайскому языку

14.12.2018 Результаты по экологии

17. 12.2018 Обновленные результаты 7-9 классов по физике и результаты 10-11 классов

17.12.2018 Результаты по итальянскому языку

17.12.2018 Окончательные результаты по английскому языку

17.12.2018 Результаты по астрономии

18.12.2018 Результаты по физической культуре

18.12.2018 Результаты по биологии

18.12.2018 Предварительные результаты 5-9 классов по математике

19.12.2018 Предварительные результаты 10-11 классов по математике

Критерии проверки работ 5-6 классов

Критерии проверки работ 7 классов

Критерии проверки работ 8 классов

Обновленные критерии проверки работ 9 классов

Критерии проверки работ 10 классов

Критерии проверки работ 11 классов

Решения задач

Показ работ и рассмотрение апелляций 

25. 12.2018 Окончательные результаты по математике


1. Требования по проведению районного этапа всероссийской олимпиады по технологии номинации «Культура дома и декоративно-прикладное творчество»

2. Требования по проведению районного этапа всероссийской олимпиады по технологии номинации «Техника и техническое творчество»


Информация о проведении апелляций:

— Апелляция по результатам районного этапа олимпиады по экологии состоится 13.12.18 (четверг) в 16.00 в ГБОУ СОШ №55 по адресу Левашовский пр., д. 5, каб. 28.

— Апелляция по результатам районного этапа олимпиады по обществознанию состоится 01.12.18 (суббота) в 15.00 в Гимназии №67 по адресу ул. Профессора Попова, д.25, каб.№30.

— Апелляция по результатам районного этапа олимпиады по английскому языку состоится 13.12.18 (четверг) в 15.30 в ГБОУ СОШ №86 по адресу ул. Мира д.4 литер А, каб. №44.

— Апелляция по результатам районного этапа олимпиады по истории состоится 16.11.18 (пятница) в 15.30 в Гимназии №67 по адресу ул. Профессора Попова, д.25, каб.№30.

— Апелляция по результатам районного этапа олимпиады по литературе состоится 20.11.18 (вторник) в 15.00 в Гимназии №610 по адресу ул. Зверинская, д.35, каб.№20.

Алгоритм подачи и рассмотрения апелляции результатов районного этапа всероссийской олимпиады школьников

График проведения районного этапа всероссийской олимпиады школьников в Петроградском районе в 2018/2019 учебном году

Распоряжение Комитета по образованию от 08.10.2018 №2926 «Об утверждении графика проведения районного этапа всероссийской олимпиады школьников в 2018-19 учебном году».

Проходные баллы для участия в районном этапе всероссийской олимпиады школьников в 2018-19 учебном году

  
Итоги школьного этапа всероссийской олимпиады школьников:

Русский язык Информатика Право
Литература Математика География
Технология (1 тур) Немецкий язык Физика
Технология (итог) Испанский язык Астрономия
ОБЖ Английский язык История
Обществознание Французский язык Химия
МХК Итальянский язык Физкультура
Экономика Китайский язык Биология
Экология


Согласие на обработку персональных данных

Полезные ссылки

  1. Методический сайт Всероссийской олимпиады школьников
  2. Сайт Центра олимпиад Санкт-Петербурга


Нормативные документы

  1. Приказ Минобрнауки от 18 ноября 2013 года №1252 «Об утверждении Порядка проведения Всероссийской олимпиады школьников (с изменениями на 17 ноября 2016 года)


Контакты

       Методисты по предметам


Ответственные за организацию и проведение Всероссийской олимпиады школьников:

      Ражева Нина Евгеньевна – методист опорного центра по олимпиадам ДДТ Петроградского района

 

Адрес эл. почты: Адрес электронной почты защищен от спам-ботов. Для просмотра адреса в вашем браузере должен быть включен Javascript.

школ Луизианы попали в заголовки газет за отправку чернокожих детей в элитные колледжи. Вот реальность.

Старшеклассник перед ноутбуком, в окружении одноклассников, одетый в школьную форму. Момент ожидания, а затем… [Ура] Эта сцена повторялась снова и снова. Студенты одной частной школы Луизианы открывают письма о зачислении из колледжей своей мечты. Видео часто становились вирусными. Этот фильм о 16-летнем студенте, поступающем в Гарвард, набрал более 8 миллионов просмотров.[Ура] Но это было еще не все. Студенты рассказали нам, что многие из их заявлений в колледж содержали ложную информацию, предоставленную школьными администраторами. И аплодисменты в этих видео прикрывали уродливую реальность жестокого обращения и запугивания в отношении Т.М. Подготовка к Ландри-колледжу. «Оскорбление эмоционально, физически. Мы поняли, хорошо. Что-то не так. Все не так с Т. Лэндри. Шестнадцатилетняя Меган Мальво — бывшая студентка. «Там был этот маленький ребенок, ему было лет 7 или 8, и он шутил в классе.Мистер Майк, он схватил ребенка за шею, поднял его и ударил всем телом о стол ». Мистер Майк — это Майкл Лэндри. Он и его жена Трейси основали школу. Его посещение стоит до 725 долларов в месяц, и оно привлекло внимание страны благодаря 100-процентному поступлению в колледж. В различных телеинтервью они выражали надежду и упорный труд. «Мы меняем общество. Мы даем надежду ». «Со щитом или на щите.» Семья Ландри отрицает фальсификацию выписок и заявлений о поступлении в колледж, а также любые обвинения в злоупотреблениях.Но они утверждают, что физические наказания назначаются, потому что они любят своих учеников и относятся к ним как к семье. Семья Ландри рассказала историю молодых чернокожих детей из рабочего сообщества, которые преодолели системные барьеры для достижения успеха. «Неважно, кто вы, неважно, где вы были в жизни, вы можете это сделать». И видео сыграли решающую роль в продвижении этого сообщения. Родители говорят, что понятия не имели, что происходит. «Все видео, я был взволнован за всех детей, потому что они дети.Маленькие черные дети вроде нас могут поступить в Гарвард или Йельский университет ». «Я имею в виду, что он хорошо говорил, и ты понял, что ты ребенок, должен пойти в эту школу. Если вы хотите лучшего для своего ребенка, отправьте его T.M. Ландри. Знаешь, ты говоришь с кем угодно в Луизиане, они тебе говорят. Когда камеры были выключены, студенты говорят, что их настраивали, допрашивали и унижали. Студенты говорят, что их также наказали физически. Иногда их заставляли часами стоять на коленях на камнях, рисе или раскаленном бетоне.«Я помню, что в первый раз меня поставили на колени, потому что он дал мне тест, и я его провалил». Расследование New York Times показало, что за кулисами Ландри неправильно заполняли стенограммы, чтобы отразить классы, которые, по словам детей, они никогда не посещали, и оценки, которые они никогда не получали. «Моя стенограмма была испорчена, потому что он испортил мой день рождения, потому что, по всей видимости, я родилась в тот год, когда была моя мама. Он проводил уроки, которые я даже не посещал, например химию ». Студенты рассказали The Times, что Ландри посоветовали им лгать в своих заявлениях в колледж о том, что они росли в семьях, отмеченных бедностью, преступностью и наркоманией.Если они откажутся, говорят, Ландри пригрозили сделать это за них. Оглядываясь назад, можно понять характер темперамента Майкла Лэндри, как в этой недавней презентации, которая переросла в всплеск страсти. [Стук кулаком] «Простите, мэм». Но это был редкий проблеск его пугающего тона. Семья Ландри производила непрерывный поток рекламных материалов, рисующих позитивную картину нетрадиционной школы. «Т. Ландри — школа без излишеств ». «Ни классных комнат, ни стен, ни книг». «Учителя без сертификатов.Классы без учителей ». После того, как видео о приеме стали набирать обороты в 2016 году, пресса со всей страны начала появляться у их дверей. «Прогуливаясь по кампусу, мы увидели этот странный стиль Лэндри». Посторонние восхищались тем, как Ландри смогли сделать невозможное. «Итак, хорошо. это невероятно.» «Результаты говорят сами за себя.» «Вперед, Ландрис…» «Они придумали секретный соус…» «Да, действительно». В интервью СМИ они называли себя семьей — «Семья прежде всего», что подталкивало детей к их академическим ограничениям.«Они сделают это. Нет, это не вариант. Отказ не вариант.» Бывшие студенты и родители рассказали нам, что во время таких посещений студенты будут вынуждены целыми днями репетировать, что сказать репортерам. «Я планирую поступить в Гарвардский университет». «Гарвардский университет.» «Стэндфордский Университет.» «Корнелл». «Коричневый.» «У нас есть книга по триггерам, как в M.I.T., так что мы просто учим друг друга». У старшеклассников был особый фокус: подготовка к экзамену ACT. Для детей младшего возраста: свободный и недостаточный учебный план, из-за которого многие классы остались позади.Когда их методы были подвергнуты сомнению, Ландри быстро отбросили любые подозрения. «Некоторым из них слегка промыли мозги». «Когда это черный ребенок, и это строго образование, с этим ребенком что-то не так». «Причина, по которой я ничего не сказал, заключалась в том, что я был напуган. Потому что все это внимание привлекали новостные каналы. Они были в статьях. Мне так промыли мозги, и я подумал, что он не может сделать ничего плохого. Но, как вы видите, поскольку я сижу здесь, я был неправ ». Теперь многие в T.Сообщество М. Ландри говорит, что они чувствуют себя обманутыми из-за нехватки времени, денег и, по иронии судьбы, образования.

Нетрадиционная математика для любознательных детей

Углубленный и подробный обзор Академии Зверей. Все, что вам нужно знать, чтобы решить, подходит ли вашему ребенку эта необычная и строгая программа по математике на дому.

Мой сын любит математику, но Академия Зверей — единственная книга по математике, которую он когда-либо умолял прочитать.

В конце концов, в большинство программ по математике не входят задачи, начинающиеся с

.

Суммарная стоимость рогатки и грейпфрута составляет $ 23…

И я, конечно, никогда не видел другой книги по математике, в которой есть ссылки на Алекса Требека, The Princess Bride, и Star Wars .(Не говоря уже о монстрах из комиксов с разными личностями, включая двухголового хранителя по имени Розенкранц и Гильденстерн.)

Но не позволяйте юмору и очаровательным мультяшным монстрам вводить вас в заблуждение. Beast Academy — это не только серьезная математическая программа: это самая сложная математическая программа, доступная для этих классов.

В этом обзоре я объясню, что отличает Beast Academy от других программ, и помогу вам решить, подходит ли она для вашей семьи.

Обзор

Beast Academy издается Art of Problem-Solving как вводный курс к их строгим учебникам для средних и старших классов для продвинутых учеников. Это комплексная программа для 2-5 классов , с четырьмя руководствами и четырьмя учебниками для каждого класса.

Руководства Академии Зверей

Руководства Beast Academy служат учебниками в этой серии, но они не похожи ни на один учебник, который я когда-либо видел. Это полноцветные книги в стиле графического романа в мягкой обложке, в которых рассказывается история четырех «зверей», посещающих Академию Зверей.Мы следим за четырьмя учениками, когда они посещают занятия, и ломаем голову над проблемами, которые ставят их наставники. У каждого из учеников и учителей есть своя индивидуальность, и весь текст пронизан юмором. На данный момент мне больше всего нравится обложка Guide 3D:

.

Это Гид 3D, а монстры смотрят… фильм в 3D!

Руководства используют множество повседневных контекстов и визуальных представлений , чтобы побудить детей глубоко задуматься и понять, что они изучают.Дети не могут просто запоминать процедуры, когда им приходится применять полученные знания в самых разных ситуациях. Например, вот три различных способа, которыми последняя глава 3B представляет свойство распределения:

Распределительное свойство применяется к площади прямоугольников. Распределительное свойство используется для определения стоимости монет на пиратском корабле. Распределительное свойство объясняется символически.

На протяжении всего руководства знаки «Стоп» в тексте напоминают детям, что им нужно остановиться и решить проблемы самостоятельно, прежде чем они прочитают ответы персонажей.В книгах часто приводится множество решений проблем, помогающих детям научиться гибко думать о математике.

Каждое руководство разделено на три главы, каждая из которых разделена на короткие разделы, которые помогают развить тему главы. Примечания в руководствах показывают, когда студент должен остановиться и заполнить соответствующие страницы в учебнике.

Практические книги

Хотя руководства знакомят с концепциями, реальная суть программы содержится в практических пособиях .Каждая глава содержит 100–150 практических задач, варьирующихся по сложности от простых практических задач до задач «двойная звезда»: сложных многоэтапных задач, которые часто требуют довольно много времени и размышлений. В целом в Beast Academy гораздо меньше основных вычислительных задач, чем в других учебных программах. Вместо этого практика базовых навыков включается в более сложные задачи.

Нет конкретных страниц обзора, но предыдущие темы постоянно пересматриваются в новом контексте. Например, периметр и площадь вводятся в 3А.Затем обе темы рассматриваются во всех трех разделах 3B и 3C (в контексте от переменных до точных квадратов) и снова являются предметом последней главы 3D.

Поддержка родителей

В Beast Academy все обучение проводится с помощью руководств и практических пособий, без руководств для учителей. Вместо того, чтобы проводить уроки, роль родителей состоит в том, чтобы обсуждать материал, а также поощрять и обучать ученика, когда он или она решает сложные проблемы. Когда мой сын учился в Beast Academy, мы всегда вместе читали руководства, останавливаясь, чтобы обсудить, пока читаем.Я часто сидел с ним, пока он решал более сложные задачи из учебника, а также сам решал их, чтобы мы могли сравнивать ответы и методы решения.

К сожалению, нет руководств для учителей для родителей, использующих Beast Academy. Тем не менее, учебники действительно предоставляют полное руководство по решениям всех практических задач, иногда даже с несколькими решениями. Для задач, отмеченных и отмеченными двумя звездами, учебники также содержат подсказки, которые помогут детям начать работу, когда они застряли.

Решение проблем

Использование Beast Academy требует совершенно другого подхода к математике, чем традиционный метод из учебников.

В некоторых отношениях этот подход более мягкий, а в других — более строгий. Это расслаблено, потому что основное внимание уделяется глубоким размышлениям о меньшем количестве проблем без большой рутинной практики. Но он более строг, потому что требует от детей напрягать мозг и применять свои знания различными способами. Это не просто типичные проблемы со словами!

Дети, которые привыкли быстро перемещаться по листу, могут сначала расстроиться, обнаружив, что им приходится работать намного усерднее и что они не могут решить все проблемы. (Родителям тоже может быть трудно смотреть — просто спросите меня, откуда я знаю!) Но обучение борьбе развивает настойчивость и установку на рост, что в конечном итоге очень хорошо окупается.

Сколько времени у Академии Зверей на обучение?

Дети обычно тратят 30-45 минут в день на решение задач. Также планируйте уделять двадцать минут один или два раза в неделю чтению и обсуждению руководства и, по крайней мере, пять минут в день, обсуждая проблемы, которые решил ваш ребенок. Если вашему ребенку нужна большая поддержка и поддержка при решении сложных проблем, это может занять больше времени.

Что мне следует использовать перед Beast Academy?

Предпосылки для Beast Academy 2A довольно просты: счет по 1, 2, 5 и 10 сверх 100, сложение и вычитание до 20, а также способность решать простые задачи со словами. (Взгляните на тест на определение уровня 2А для получения более подробной информации.) Практически любая формальная программа детского сада и первого класса подготовит вашего ребенка с этими навыками, поэтому прочитайте мои обзоры других отличных программ домашнего обучения, чтобы найти ту, которая лучше всего подойдет вашему детсадовцу. или первоклассник.

Какой тип ученика преуспевает в Beast Academy?

Beast Academy отлично подходит для детей, которые хорошо успевают по математике, но быстро устают от повторяющихся занятий. Программа требует терпения и настойчивости для решения сложных задач, но также помогает развить терпение у детей, которым это не дается естественным путем.

Я не рекомендую Beast Academy для детей, которые плохо разбираются в математике. (и авторы тоже). Если вы думаете, что стиль комиксов понравится вашему ребенку, но вы не уверены, что он станет отличной основной учебной программой для Вашему ребенку Гиды станут отличным дополнением.

Сколько стоит Академия Зверей?

Годовые руководства и учебники (по четыре каждого) стоят 108 долларов. Никаких манипуляторов или других ресурсов не требуется.

Если Beast Academy не кажется развлечением для вас и вашего ребенка, не используйте ее! Существует множество других отличных учебных программ по математике, построенных в более традиционном формате. Просмотрите мою учебную программу , страницу , где вы найдете обзоры других моих любимых программ, чтобы помочь вам найти ту, которая подходит для вашей семьи.

Но если ваш ребенок любит решать интересных, наводящих на размышления проблем и преуспевает, когда концепции представлены с большим количеством контекста и визуальных элементов, Академия зверей может ему подойти. Несмотря на мультяшные монстры и шутки, именно тщательная и строгая программа хорошо готовит детей к более высокому уровню математики.

Обновлено в мае 2020 года. Это мое честное мнение о программе; Мне не платили и не компенсировали никакую компенсацию за обзор.

Некоторые ссылки в этом посте могут быть партнерскими.Если вы покупаете товар по партнерской ссылке, я могу получить комиссию без каких-либо дополнительных затрат для вас.

Обратите внимание, что комментарии к этому сообщению закрыты. Если у вас есть вопрос, вы можете связаться со мной здесь.

олимпиад НИУ ВШЭ. Всероссийская олимпиада «Высшая проба

«.

Пензенский государственный университет — соорганизатор Всероссийской олимпиады школьников «Высший тест» включенных в Перечень олимпиад школьников Минобрнауки России на 2019-2020 учебный год… Эти олимпиады дают победителям и призерам право претендовать на одну из льгот при поступлении в российские вузы (зачисление без вступительных экзаменов или 100 баллов по основному предмету ЕГЭ). Привилегия должна быть подтверждена при приеме сдачей экзамена по профилю олимпиады не менее 75 баллов. Олимпиада проводится в два тура (заочный и очный).

Олимпиада « Высший стандарт » проводится по общеобразовательным предметам: биология, востоковедение, восточные языки, дизайн, журналистика, иностранные языки, информатика, история, история мировых цивилизаций, культурология, математика, обществознание, основы бизнеса, политология, право, психология, русский язык, социология, физика, филология, философия, финансовая грамотность, химия, экономика, электроника и вычисления.

Участие в олимпиаде бесплатное. Олимпиада проводится в два этапа. Для участия школьники должны пройти процедуру удаленной регистрации на сайте в установленные сроки. Отборочный этап проходит в заочной форме в режиме онлайн-тестирования. Вы можете принять в нем участие с любого компьютера, имеющего доступ в Интернет. Финальный этап проводится в очной форме более чем в 30 городах Российской Федерации, стран СНГ и Балтии.

В разделе «Регистрация на второй этап» выберите олимпиады, в которых вы примете участие (если вы прошли несколько олимпиад) и во втором этапе. Если вы столкнулись с проблемами при входе в личный кабинет, ознакомьтесь с инструкциями по решению технических проблем. Если вы забыли свое имя пользователя и пароль, воспользуйтесь процедурой «Сменить пароль» на странице входа. .

После этого вы получите электронное письмо с подтверждением вашего выбора, и оно будет доступно для скачивания в вашем личном кабинете. Титульный лист , который необходимо распечатать и взять с собой в день соревнования. Оргкомитет не предоставляет участникам возможность распечатать титульный лист на месте проведения соревнований Олимпиады.

Если вы не выбрали город участия в указанные сроки, желательно прибыть на место проведения заранее в день соревнований. Организаторы позаботятся о том, чтобы вас пригласили на второй этап, и определят аудиторию, в которой вы будете выполнять задания олимпиады.

Убедительно просим вас также проверить свои личные данные — фамилия, имя, отчество, дата рождения, информация об учебном заведении, в котором вы учитесь. Победителем олимпиады может стать любой из вас. В дипломе, свидетельствующем о ваших успехах, будет указана информация, которую вы указали в личном кабинете. В ваших интересах четко представлять все, чтобы у вас не было проблем с внесением изменений. В частности, обратите внимание, что информация об образовательном учреждении совпадает с той, которая будет указана в справке (допускаются общеупотребительные сокращения).

Собираемся на олимпиаду

Приехать на соревнования олимпиады нужно заранее (20-30 минут), не стоит рассчитывать, что вы появитесь за 5 минут до старта и все успеете. Подумай о времени! Если вы опоздали на старт олимпиадного соревнования, вы будете допущены к нему, но время выполнения заданий продлено не будет.

Участник должен иметь при себе:

  • Документ, удостоверяющий личность (паспорт, свидетельство о рождении)
  • Документ о статусе студента

Для учащихся образовательных учреждений — справка из школы
— для студентов колледжей — справка о продолжении обучения по общеобразовательным программам среднего (полного) образования

Обращаем ваше внимание, что по окончании каждого конкурса второго (финального) этапа участник должен предоставить справку из школы / колледжа вместе с работой.Если вы участвуете в олимпиадах по нескольким предметам, то вам потребуется несколько экземпляров справок , удостоверяющих статус студента. Разрешается иметь оригинал свидетельства и необходимое количество фотокопий оригинала свидетельства. В этом случае оригинал сертификата предоставляется организаторам олимпиадных соревнований, которые, убедившись в наличии сертификата, принимают его ксерокопию.

  • Обложка (печать)
  • Ручка с черными или синими чернилами

Питьевую воду можно брать с собой.

На олимпиадах по физике и электронике разрешается пользоваться простейшими калькуляторами.

На олимпиаду брать нельзя:

  • справочные материалы
  • свой доклад (черновик вам передадут организаторы)
  • карманные компьютеры и прочие электронные вычислительные устройства
  • мобильные телефоны и другие средства связи
  • игроков
  • калькуляторов (за исключением : олимпиада по физике и электронике)
  • технические средства прочие

Использование этих материалов и средств не допускается как в классе, так и во всем здании на протяжении всего соревнования до окончания времени, отведенного для выполнения олимпиадного задания.

Перед стартом

При входе в здание необходимо оставить верхнюю одежду в гардеробе.

Далее следует внимательно ознакомиться с информацией, размещенной для участников олимпиады. Уверяем, что ответы на большинство вырванных из груди вопросов вы получите, если все внимательно прочитаете. Найдите свою фамилию в списке участников, отсортированном по классу и алфавиту. Напротив — номер аудитории, в которой вы будете выполнять задание.Никакой дополнительной регистрации участники олимпиады не проходят, поэтому смело отправляйтесь прямо к зрителям.

За 30 минут до начала конкурса, прямо у входа в зрительный зал начнется регистрация участников, пришедших для участия в конкурсе. При входе в зал необходимо предъявить паспорт или другой документ с фотографией, удостоверяющей вашу личность, справку из школы, чтобы организаторы удостоверились, что у вас есть необходимые документы, и назовут вашу фамилию.

Если вы, несмотря на предупреждение, пришли на соревнования без документов, лучше сразу убедиться, что до окончания соревнований олимпиады необходимые документы были доставлены вам родителями или друзьями. Вы можете быть допущены к участию в конкурсе, но выполненные вами работы будут приняты для проверки только при предъявлении всех необходимых документов.

По предложению организаторов оставьте личные вещи в специально отведенном для них месте в зале.Заранее отключите все устройства, которые могут внезапно издавать громкие звуки во время гонки (например, мобильный телефон).

В начале

Участники занимают указанные места, организаторы раздают бланки ответов.

Все пустые поля на титульном листе должны быть заполнены.

После завершения подготовительных процедур организаторы раздадут бланки заданий лицевой стороной вниз (чтобы текст не был виден), которые можно перевернуть только по сигналу организаторов.Этот момент считается началом соревнований олимпиады. Время начала и окончания соревнований олимпиады будет указано на доске.

Во время конкурса

Работайте с вниманием и сосредоточенностью.

Не нарушать правила участия в олимпиадных соревнованиях. Не рискуй! За наличие на рабочем месте запрещенных предметов вам будет запрещено участвовать в конкурсе, независимо от того, использовали вы их или нет.

Мы рекомендуем использовать черновик по назначению и не пытаться сначала проделать работу над черновиком, чтобы потом перенести результаты в окончательную копию — времени может не хватить. Помните, что черновики не рассматриваются (но должны быть отправлены). Шашки не проверяются, за исключением проектов работ участников олимпиад по математике, физике, дизайну, востоковедению и литературе , указавшим «см. Черновик» в форме ответа. Содержание черновиков не может быть основанием для апелляции.

Недопустимо делать какие-либо особые отметки на листе ответов, устанавливающих авторство, особенно писать фамилию, имя, отчество.

Во время проведения конкурса участник имеет право обращаться к организаторам с вопросами по организации конкурса и оформлению работы, вносить замечания и жалобы по заданиям олимпиады в протоколе конкурса, обращаться за медицинской помощью, подавать работать с опережением графика, а текст олимпиадного задания получить по окончании конкурса.

При необходимости вы можете покинуть аудиторию на 5-7 минут с разрешения и в сопровождении организаторов (предупреждение: выходить из аудитории в течение первых 60 минут можно только в случае крайней необходимости; исключение: вход и выход из аудитории во время прослушивания запрещен. ), оставляя в классе есть бланк с заданиями, бланками ответов и т.д. Менее чем за 15 минут до окончания конкурса нельзя покидать аудиторию, чтобы не шуметь и не мешать другим участникам при выполнении задание.

Во время конкурса также не допускается:

  • задавать вопросы другим участникам и отвечать на вопросы других участников
  • встать и пересаживаться
  • обманывать и позволять другим участникам обманывать
  • обменять любые материалы и предметы
  • продолжить работу по истечении времени, отведенного на олимпиадное задание

Правила оформления работы

    Участник должен завершить работу на листе ответов.Первый лист листа ответов — это протокол участия, второй лист — протокол рабочего теста, затем листы для записи решений и ответов на задания олимпиады, на них можно делать пометки с обеих сторон. Запрещается использовать текст задания в качестве формы ответа.

  1. Участник должен заполнить титульный лист, заполнить и подписать Протокол участия (первая страница формы ответа), заштриховать овал, соответствующий классу в Протоколе тестирования производительности (вторая страница формы ответа), и выполнить больше ничего не пишите на первой и второй страницах формы ответа.
  2. Работа должна выполняться синей или черной ручкой. Запрещается пользоваться ручкой с красными и зелеными чернилами, использовать карандаш для записи решений и ответов.
  3. Черновик и дополнительные листы к форме ответа предоставляются организаторами по запросу участников.
  4. Шашки не проверяются, за исключением проектов работ участников олимпиад по математике, физике, дизайну, востоковеду и литературе, указавших в форме ответа «посмотреть черновик».
  5. В форме ответа и в проектах, представленных на проверку, нельзя указывать ФИО, делать какие-либо записи с указанием авторства работы.
  6. В лист для ответов можно внести исправления, которые должны быть четкими и недвусмысленными. Если необходимо внести исправления, то следует внимательно вычеркнуть неправильный ответ и написать правильный.
  7. Почерк участника должен быть четким. Жюри может отказать участнику в проверке работы в случае «нечитаемого» почерка.
  8. Допускается замена пера (при наличии соответствующей отметки в форме), титульного листа, формы ответа.
  9. Участники олимпиады по информатике выполняют работу на компьютере, вводя ответ на задание или программный код в систему тестирования, получив от организатора логин и пароль.

В конце

За 15 минут до окончания олимпиады соревнований организаторы предупредят вас, что срок выполнения работ подходит к концу.Перенесите решения и ответы из проекта в лист ответов, если вы еще этого не сделали.

По истечении отведенного времени необходимо прекратить выполнение задания и передать организаторам:

  • листов ответов
  • текст задания (при досрочном сдаче работы)
  • справка из школы / колледжа
  • согласие на обработку персональных данных и публикацию работы
  • чертежей для последующего уничтожения (чертежи участников олимпиады по математике, физике, востоковедению и литературе можно проверить при необходимости, к работе участника прилагаются чертежи по литературе и востоковедению)

При подаче работы необходимо предъявить организаторам документ, удостоверяющий личность (паспорт или свидетельство о рождении) и дать справку из школы / колледжа. Напоминаем еще раз, что выполненная вами работа будет принята на проверку только при предъявлении всех необходимых документов.

Не вставайте со своего места, пока организатор в аудитории не разрешит вам это сделать!

Вы можете оставаться в зале, пока организаторы пересчитывают и упаковывают работы участников.

Желаем уверенности в своих силах, высокой результативности и успехов в олимпиадных соревнованиях!

В 2018-2019 годах состоится очередная межрегиональная олимпиада школьников «Высшее испытание», учрежденная Высшей школой экономики (НИУ ВШЭ).Этот популярный конкурс проводится с 1998 года. Современное название он получил в 2012 году. Бесплатно могут принять участие старшеклассники из России и других стран «постсоветского пространства», в том числе Литвы, Латвии и Эстонии. Победители и призеры олимпиады имеют право на значительные льготы при поступлении в российские вузы.

Миссия олимпиады

Помимо общекультурной миссии популяризации образования среди молодежи, «Высшая проба» стремится отбирать лучших студентов для ведущих вузов страны. Такие конкурсы частично заменили предыдущие вступительные экзамены и являются реальной альтернативой ЕГЭ. Практически все крупные российские университеты отдают приоритет победителям и призерам своих олимпиад (или других подобных соревнований) при зачислении. «Высший стандарт» отличается гуманитарной направленностью. Среди основных дисциплин для старшеклассников в 2018-2019 годах — экономика, английский язык, философия и другие социальные науки.

Подготовка к участию

Для подготовки к олимпиаде «Высший тест» вы можете использовать демонстрационные версии тестов, а также задания и темы из прошлых лет.Все они доступны бесплатно на официальном сайте https://olymp.hse.ru/mmo/. «Высший стандарт» — престижный, высококонкурентный конкурс, поэтому участие в нем имеет смысл только в том случае, если вы действительно заинтересованы в выбранной теме и обладаете необходимыми знаниями. Следует добавить, что такие соревнования требуют физической выносливости и устойчивости к стрессовым ситуациям.

Учетные и организационные вопросы

Десятки тысяч школьников из России и зарубежных стран участвуют в олимпиаде «Высший тест».Количество призов — около двух тысяч. Регистрация осуществляется в электронном виде на официальном сайте до октября 2018 года. Сам конкурс разделен на два этапа. Первый тур запланирован на ноябрь-декабрь 2018 года. Он отборочный. Экзамен проводится заочно, онлайн с любого компьютера, при условии ввода имени пользователя и пароля, предоставленных организаторами. По некоторым предметам участников попросят написать короткое эссе. Сроки проведения отборочного тура определяются заранее, время выполнения заданий строго ограничено.Подробная информация об этом будет опубликована в сентябре 2018 года. Победители прошлогоднего «Высшего теста» из не выпускных классов освобождаются от квалификационных соревнований, но также должны зарегистрироваться. Желающие могут участвовать в олимпиадах сразу по нескольким предметам, если позволяет график.

Участники, успешно прошедшие первый раунд, допускаются ко второму, финальному раунду. Он будет проходить в январе-феврале 2019 года только в очной форме в разных городах России или на территории зарубежных стран.Организационную и техническую поддержку на данном этапе оказывают местные учебные заведения, выбранные в качестве партнеров по НИУ ВШЭ.

Все заявители должны иметь при себе ряд документов:

  • Печатная «Титульная страница», отправляемая по электронной почте. Подтверждает право на участие в финальном конкурсе.
  • Документ, удостоверяющий личность (паспорт, свидетельство о рождении) и справка из учебного заведения.

Школьники могут самостоятельно выбрать удобное для них место и дату проведения конкурса из предложенных организаторами вариантов.Во время финала запрещается использование любых справочных материалов, а также любых электронных устройств. В некоторых случаях для простейших калькуляторов допускаются исключения.

После проверки работ и возможных обращений жюри объявит победителей и призеров по всем предметам. Обычно это происходит ранней весной. Позже в Москве и других городах России, где открыты филиалы НИУ ВШЭ, состоится церемония награждения с вручением памятных дипломов.Эти документы можно подавать в приемные комиссии вузов России.

Победитель получает все?

«Высший тест» и другие подобные соревнования часто критикуют за «элитарность» и «дискриминацию» студентов, не имеющих возможности или желания участвовать в них. В первую очередь это касается детей из малообеспеченных семей или из отдаленных регионов Российской Федерации. Иногда упоминаются случаи мошенничества, выявленные в ходе этих конкурсов.Однако такое случается редко. Фактические данные свидетельствуют о том, что лауреаты олимпиад прошлых лет в целом лучше учатся в вузах, чем студенты, зачисленные по системе ЕГЭ.

Большинство предметов «наивысшего стандарта» включены в официальный «Список олимпиад школьников», утвержденный Правительством Российской Федерации. Победители и призеры данного конкурса имеют право на следующие льготы:

  • зачисление в вузы без дополнительных проверок знаний по профильному предмету олимпиады;
  • максимальный балл ЕГЭ по профильному предмету олимпиады;
  • студентов из неполных классов могут принять участие в финальном туре в следующем году, не проходя через сито предварительного отбора.

Ежегодно более тысячи лауреатов «Высшей пробы» становятся студентами самых престижных вузов России.

Государственное бюджетное образовательное учреждение «Центр поддержки одаренных детей» информирует об интеллектуальных олимпиадах, проводимых Высшей школой экономики в 2017-2018 учебном году.

Межрегиональная олимпиада школьников «Высший стандарт»

Олимпиада, на которую приглашаются ученики 7-11 классов, проводится по 22 профилям.Девять олимпиад в рамках «Высшего теста» удостоены I уровня, восьми — II уровня, трех — III уровня (Приказ Минобрнауки России от 30.08.17 № 866.).

В Высшей школе экономики, как и в других вузах, уже утверждены правила приема на олимпиады. Как правило, льготы предоставляются учащимся, окончившим 11-й класс в 2018 году, но по некоторым образовательным программам преференции распространяются также на школьников, окончивших школу в прошлом году, когда они были 10-ми классами.Кроме того, победители и призеры 7-10 классов следующего года сразу же приглашаются на финальный этап, минуя отборочный тур.

Регистрация: со 2 октября по 20 ноября https://olymp.hse.ru/mmo/2017/instr-reg/

Расписание межрегиональной олимпиады школьников «Высшая выборка» на 2017/18 учебный год.

Первый (отборочный) заочный этап

Второй (заключительный) очный этап

Конкурс научно-исследовательских и проектных работ школьников «Высший пилотаж»

График конкурса научно-исследовательских и проектных работ школьников «Высший пилотаж» на 2017/18 учебный год

Регистрация и представление работ участниками 11 декабря 2017 г.
Окончание регистрации и сдача работ 12 марта 2018 г.
Окончание регистрации и сдача работ «Дизайн» 2 апреля 2018 г.
Окончание регистрации и сдача работ в Интел-Авангард 5 февраля 2018
Отправка приглашений участникам Intel-Авангард до 15 февраля 2018 г.
Публикация результатов конференции «Интел-Авангард» 12 марта 2018 г.
Конференция Интел-Авангард (Вороново) 22-25 февраля 2018 г.
Сроки проведения первого этапа 11 декабря 2017 г. — 12 марта 2018 г.
Проверка работ 13-30 марта 2018 г.
Публикация результатов первого этапа 6 апреля 2018 г.
Дата второго этапа 21-22 апреля 2018 г.
Публикация результатов конкурса 30 апреля 2018 г.

Олимпиада НИУ ВШЭ для студентов и выпускников

Олимпиада проводится по широкому кругу профилей, соответствующих образовательным программам магистратуры Высшей школы экономики.К участию в олимпиаде приглашаются студенты, обучающиеся по образовательным программам бакалавриата или специалитета, а также выпускники образовательных учреждений высшего профессионального образования независимо от гражданства. Дипломанты олимпиады получают преференции при поступлении в НИУ ВШЭ на магистерские программы, соответствующие профилю олимпиады.

Расписание олимпиады НИУ ВШЭ для студентов и выпускников на 2017/18 учебный год

Олимпиада «Ступени»

К участию в олимпиаде приглашаются учащиеся 9-11 классов из всех регионов Российской Федерации, имеющие опыт написания научных работ или знающие, как «организована» научно-исследовательская деятельность и научная работа.

Олимпиада проводится в два этапа: заочный и очный. На заочном этапе представлены задания тестового типа на индивидуальные навыки исследователя: такие как умение ставить цель и задачи, формулировать исследовательскую гипотезу, планировать ход и структуру работы и др. На очном этапе , участникам олимпиады будет поручено провести экспертную оценку исследовательской работы по критериям и написать рецензию на эту работу.

Победители олимпиады получат сертификат HSE и призы.

Организаторами олимпиады являются лицей НИУ ВШЭ совместно с Дирекцией профориентации и работы с одаренными учениками.

Подробнее Опубликовано 29.09.2016 19:26 Просмотров: 1272

Национальный исследовательский университет «Высшая школа экономики» сообщает о проведении олимпиады «Высший тест».

К участию в олимпиаде приглашены ученики 7-11 классов из России и ближнего зарубежья. Олимпиада «Высший тест» включена в Перечень олимпиад для школьников, предоставляющих льготы при поступлении в высшие учебные заведения Российской Федерации.

Соревнования первого (отборочного) этапа олимпиад пройдут в заочном формате (онлайн) конец ноября — начало декабря 2016 года … Только зарегистрированные участники. Отборочный этап проходит в форме заочного тестирования строго в указанное время. Вы можете участвовать в каждом конкурсе только один раз.

Февраль 2017 г. — второй (заключительный) этап

Олимпийские соревнования второго (финального) этапа будут проводиться в очной форме.Список городов-организаторов будет утвержден после определения результатов отборочного этапа. Обращаем внимание всех школьников города Иваново Ивановской области: по результатам участия детей в заочном этапе. , город Иваново может претендовать на право проведения очного этапа олимпиады «Высший тест».

Участие в олимпиаде бесплатное.

Предметов олимпиады:
7 класс — иностранные языки, история, литература, математика, русский язык;
8 класс — иностранные языки, история, литература, математика, русский язык, обществознание, экономика;
9-11 классы — востоковедение, восточные языки, дизайн, журналистика, иностранные языки, информатика, история, история мировых цивилизаций, литература, математика, обществознание, право, психология, русский язык, физика, экономика, электроника.

48 спортсменов, которых стоит посмотреть на Олимпийских играх в Токио

Олимпийские игры в Токио не будут похожи на Игры, которые вы смотрели в прошлом. Спортсмены будут соревноваться без зрителей, даже без своих семей, которые могли бы подбодрить их, среди других ограничений. Эти строгие правила на местах будут действовать из-за пандемии COVID-19, которая только что вызвала новое чрезвычайное положение в Токио. Но глобальное событие продвигается вперед, и около 11000 участников направляются в Токио на Игры.Такая массовая группа элитных спортсменов обязательно принесет захватывающие соревнования, захватывающие соперничества и моменты для книг рекордов.

Вот 48 часов, на которые стоит посмотреть, когда Игры официально начнутся 23 июля.

(Щелкните каждый значок плюса, чтобы узнать больше.)

Simone Biles


Gymnastics, U.С.

Симона Байлз соревнуется на бревне во время соревнований по классической гимнастике GK в США 22 мая 2021 года в Индианаполисе, штат Индиана.

Эмили Чинн — Getty Images

Когда в марте величайшая гимнастка всех времен узнала, что все честолюбивые олимпийцы боятся — что Игры отложены на год, — это сильно ударило.

«Я злилась, мне было досадно, мне было грустно», — говорит Симона Байлз, действующая олимпийская чемпионка по гимнастике в многоборье. «Я должен был позволить себе почувствовать все эмоции. Я на 100% чувствовал, что не планировал этого — теперь мы должны вернуться к чертежной доске и разобраться с множеством переменных. Но, в конце концов, это больше, чем я. Мы должны убедиться, что все в мире в безопасности ». Тем не менее, добавляет она, «это было отстой».

Подробнее

Тренировки Zoom с тренерами и товарищами по команде, прогулки с собаками и чередование приступов уборки с запоями Netflix помогли ей пережить два месяца, когда ее тренажерный зал был закрыт. Когда ей разрешили вернуться на модифицированные тренировки, ее новые тренеры, Сесиль и Лоран Ланди, сосредоточились на основных принципах, чтобы сделать и без того непревзойденного Байлза еще лучше.

Непобедимая в соревнованиях по женской гимнастике с 2013 года в многоборье Байлз по сути будет соревноваться с собой в Токио. Ее фирменные навыки, которые проверяют пределы человеческих способностей и гравитации, набирают достаточно очков сложности, поэтому она часто намного опережает своего ближайшего конкурента.В самом деле, для всех, кроме горстки соперников, соревнование, по сути, окончено еще до того, как начнется.

Байлз планирует войти в историю Токио как первая гимнастка, выполнившая опасно сложный прыжок, который до сих пор на Играх выполняли только мужчины. Это на вершине соскока с бревна, названного в ее честь, который настолько рискован, что Международная федерация гимнастики не наградила его высоким значением сложности, чтобы отговорить других гимнасток от попытки его и потенциально травмировать себя, а также двумя бросающими вызов гравитацией навыками акробатики, которые имеют становятся визитной карточкой ее распорядка на полу. Лэндисы говорят, что Байлз не гонится за медалями, а только проверяет себя, чтобы увидеть, как далеко она — и гимнастика — могут зайти.

В то время как сплоченная семья Байлз не приедет в Токио, чтобы подбодрить ее из-за ограничений COVID-19, они и десятки молодых гимнасток, которые тренируются с ней в спортзале ее родителей в Спринг, штат Техас, планируют ночевать и смотреть вечеринки, как никто другой, в дни ее соревнований. Несколько сотен человек соберутся в тренажерном зале, чтобы поесть, устроить торжество и посмотреть прямую трансляцию событий в Токио.«У нас будет огромный экран и прямая трансляция; это будет потрясающе, — говорит ее мама Нелли. Шоу должно быть потрясающим для просмотра даже без особого напряжения. —Алис Парк

Ариарн Титмус

Плавание, Австралия

Когда Ариарн Титмус на соревнованиях по плаванию в Австралии на дистанции 400 м вольным стилем заняла второе место в истории среди женщин, наблюдатели назвали это «предупредительным выстрелом» в адрес U. S. Powerhouse Кэти Ледеки, установившая мировой рекорд на Олимпийских играх в Рио.

Подробнее

Титмус уже показала, что способна победить Ледеки; На чемпионате мира 2019 года она победила американского пловца на дистанции 400 м вольным стилем, хотя Ледеки выбыла из двух гонок на этом мероприятии из-за болезни. Соперничество между командами Австралии и США по плаванию, ставшее ярким событием летних Олимпийских игр на протяжении десятилетий, снова будет продемонстрировано в Токио.На испытаниях 20-летний Титмус по прозвищу Терминатор предсказал: «Я думаю, что Олимпийские игры не станут достоянием Америки». — Эми Гуния

Алекс Морган

Футбол, США

Игрок «Тоттенхэм Хотспур» Алекс Морган на разминке перед матчем Континентального кубка в лондонском Медоу-парке.

Адам Дэви — PA Images / Getty Images

Если у женской национальной сборной США по футболу и есть последняя цель, то она такова: стать первой женской командой, когда-либо выигравшей чемпионат мира и Олимпийские игры. Алекс Морган, пятый лучший бомбардир в истории команды, будет ключом к предотвращению еще одного разочарования: прошлое У.S. Победы на чемпионатах мира в 1999 и 2015 годах сменились олимпийскими разочарованиями.

Подробнее

Первоначальный план Морган в отношении Токио состоял в том, чтобы играть через несколько месяцев после рождения ее первого ребенка, дочери Чарли, в мае 2020 года. С переносом Игр Морган смогла постепенно выйти в лучшую форму к ее третьим Олимпийским играм; она забила гол в игре против Франции и была названа игроком месяца Национальной женской футбольной лиги в мае после того, как забила гол в каждой из первых четырех игр с «Орландо Прайд» в этом сезоне. — Шон Грегори

Caeleb Dressel

Swimming, U.S.

На террасе у бассейна нет пропавших без вести Caeleb Dressel. Его левую руку покрывает чернильный рукав, в котором запечатлены те качества, которые делают 24-летнего флоридца лучшим пловцом, за которым стоит наблюдать на различных соревнованиях в Токио.

Подробнее

Начиная с его плеча, орел с распростертыми крыльями парит на рычащем медведе, который сидит над обнажающим зубы аллигатором (дань уважения его альма-матер, Университету Флориды).Дрессел называет их своими «духовными животными», от которых он передает силу, свирепость и скорость, благодаря которым он завоевал исторические восемь медалей на чемпионате мира 2019 года (это на одну больше, чем рекорд Фелпса на этом мероприятии). Мировой рекордсмен в беге на 100 м баттерфляем имеет шанс завоевать семь медалей в Токио. —Алис Парк

Эллисон Феликс

Легкая атлетика, U.С.

Фотография Дженеба Адуайома для TIME

Получите распечатку обложки токийской Олимпиады TIME с Эллисон Феликс здесь

Эллисон Феликс, выигравший в Токио еще одну медаль, уже выиграл больше золотых медалей чемпионатов мира, чем любой другой легкоатлет за всю историю — станет самой титулованной легкоатлеткой в ​​истории Олимпиады.И теперь ее влияние выходит далеко за рамки ее записей. — Шон Грегори

Габриэль Медина

Серфинг, Бразилия

Возможно, нет никого лучше, чем этот 27-летний бразилец, чтобы влюбить спортивные массы в серфинг в его олимпийском дебюте. Лучший серфер в мире, Габриэль Медина, известен своим захватывающим акробатическим стилем, часто взлетая над волнами для высокотехничных маневров.

Подробнее

В 2016 году он стал первым серфингистом, совершившим сальто на своей доске над водой на соревнованиях. Медина выросла на южном атлантическом побережье Бразилии, где волна неизменно велика. Это очень далеко от того, с чем он столкнется на пляже Цуригасаки в Японии. «[Волны] маленькие и необычные», — сказал он Guardian в мае.«Но если ты хочешь быть лучшим, ты должен делать все в любых условиях». — Сиара Ньюджент

Кевин Дюрант

Баскетбол, США

Кевин Дюрант из «Бруклин Нетс» надеется ударить против «Бостон Селтикс» в пятой игре первого раунда плей-офф НБА 2021 года на стадионе «Барклайс-центр» 1 июня 2021 года в Бруклине, штат Нью-Йорк.

Стивен Райан — Getty Images

Несмотря на его героические усилия во втором раунде плей-офф НБА в этом году — он набрал 48 очков и отыграл все 53 минуты в овертайме 7-й игры, проиграв «Милуоки Бакс» — Кевин Дюрант не смог довести свою звездную команду Brooklyn Nets до чемпионата. .

Но ему еще нужно собрать оборудование этим летом.

Подробнее

Дюрант решил сыграть в Токио, несмотря на то, что в прошлом сезоне пропустил 37 игр из-за травм и проблем с COVID-19, а также весь сезон 2019–20 из-за травмы ахилла. Но привлекательность третьего золота подряд — Дюрант был лучшим бомбардиром сборной США как в Лондоне, так и в Рио — очень сильна. Он также воссоединится со своим бывшим тренером GoldenState Warriors Стивом Керром, который будет помощником тренера на Олимпийских играх, и бывшим товарищем по команде Дреймондом Грином. Многие из самых известных игроков НБА обошли Токио, но Дюрант закрепит за сборной США, которая по-прежнему остается сильным фаворитом на победу. —Шон Грегор и

Эмили Сибом

Плавание, Австралия

Пятикратный призер Олимпийских игр отправляется на свои четвертые Игры с новой целью за пределами бассейна. В январе 29-летняя Эмили Сибом рассказала, что боролась с расстройством пищевого поведения в течение двух лет, заявив в социальных сетях, что переедание, чистка и постоянное взвешивание были частично вызваны давлением, которое, по ее мнению, «единственный способ плавать быстрее — это проигрывать. масса.”

Подробнее

Сибом в своем посте пообещала «подарить моему телу любовь, которую оно заслуживает», а в июне она заняла второе место в беге на 100 м на спине на олимпийских соревнованиях в Австралии. Сибом не уточнила, что стоит за ее собственным расстройством пищевого поведения, но перед национальными испытаниями «Плавание Австралии» обвинили в ядовитой культуре, особенно в отношении женщин. Специалист по бабочкам Мэдлин Гроувс отказалась от участия в мероприятии, сославшись на «женоненавистников», которые «стыдятся тела или с медицинской точки зрения выделяют газ [молодые женщины и девушки].«Австралийские официальные лица по плаванию заявили, что они расследуют заявления Гроувса. —Алис Парк

Хенд Заза

Настольный теннис, Сирия

Сирийская теннисистка Хинд Заза играет во время чемпионата местных клубов 13 марта 2020 года в Дамаске, Сирия.

Фото Луай Бешара — AFP / Getty Images

Хенд Заза, которому всего 12 лет, станет самым молодым спортсменом, участвовавшим в соревнованиях в Токио, и одним из самых молодых спортсменов, когда-либо прошедших квалификацию на Игры. Это возможно, потому что настольный теннис, в отличие от некоторых олимпийских видов спорта, не имеет возрастных ограничений.

Подробнее

Молодой сириец обыграл 42-летнего соперника в прошлогоднем квалификационном турнире Западной Азии к Олимпийским играм и вышел на Игры, заняв 155-е место.Ее появление в Токио также станет важной вехой для ее страны: Сирия ни разу не участвовала в Играх в настольный теннис через квалификацию.

Заза малоизвестен на элитном уровне спорта, где доминируют игроки из Китая, Южной Кореи, Японии и Германии. Ее тренер сказал, что из-за гражданской войны, которая опустошала страну более десяти лет, Заза не может участвовать во многих турнирах. Хотя ее шансы на продвижение в Токио невелики, само ее присутствие — триумф. —Райса Брунер

Ханна Робертс

BMX, США

Гонки на BMX являются частью летних Олимпийских игр с 2008 года, но соревнования по фристайлу дебютируют в Токио. Ханна Робертс, действующая в настоящее время чемпионка мира среди женщин, стала первой женщиной, которая приземлилась на 360-градусном хайвипе на соревнованиях и выиграла все три этапа Кубка мира в 2019 и 2020 годах.

Подробнее

Она начала кататься на велосипеде в 9-летнем возрасте в Мичигане — и в начале тренировки она сломала позвонок. После выздоровления она приняла участие в своем первом соревновании. Сейчас

19, Робертс так же уверен, как и сторонник справедливой оплаты труда в спорте, указывая на неравенство в призовых и спонсорских возможностях. Она надеется, что Олимпийские игры, на которых Робертс является фаворитом золота, помогут решить эту проблему. —Райса Брунер

Сью Бёрд

Фотография Паолы Кудацки для TIME

Получите распечатку обложки токийской олимпиады TIME с изображением Сью Берд здесь

Сью Берд, 40 лет, взяла на себя роль иссохшего ветерана, как на корте, так и за его пределами.Теперь Бёрд, не знающий себе равных по голевым передачам WNBA, продемонстрирует свое необычное видение пола на Олимпийских играх в Токио, где она и ее товарищ по команде Дайана Таурази надеются стать первыми баскетболистками, выигравшими пять олимпийских золотых медалей. Бывший товарищ по команде Стейли теперь их тренер. — Шон Грегори

Карисса Мур

Серфинг, США

Это бурные времена для соревновательного серфинга.Этот вид спорта, который давно считается аутсайдером, требует серьезного атлетизма и высокотехнологичных навыков, которые будут продемонстрированы всему миру, когда серфинг дебютирует на Олимпийских играх в Токио. Карисса Мур, уроженка Гавайев и четырехкратная чемпионка Мировой лиги серфинга (WSL), является фаворитом, выигравшим это первое золото.

Подробнее

Это связано с тем, что арсенал мультиваток Мура — поворотов, захватов, поворотов и других маневров, выполняемых над гребнем волны — не имеет себе равных.Во время апрельского турнира в Австралии Мур завершила крупнейшее в своей карьере доминирующее событие: ее доска поднялась над волной, когда она повернула ее, и схватила ее левой рукой, прежде чем чисто приземлиться в воде. Воздушный реверс принес ей 9,9 из 10. Она в недоумении схватилась за голову обеими руками. «Это было так здорово, потому что я не думал об этом», — говорит Мур. «Просто так получилось.»

«Она из тех, кто сломал барьеры того, что можно делать на волне», — говорит Джесси Майли-Дайер, в прошлом профессиональный серфер, которая сейчас курирует соревнования WSL.«Это ориентир для современного серфинга. Это так важно.

Этот вид спорта также является лидером в области равной оплаты труда мужчин и женщин, что является труднодостижимой целью для многих олимпийских спортсменов. В сентябре 2018 года WSL объявил, что будет предлагать одинаковые призовые деньги мужчинам и женщинам в туре. Это большой скачок по сравнению с 2010 годом, когда Мур был новичком, когда чемпион мира среди мужчин получил премию в размере 100 000 долларов, а чемпионка среди женщин — 30 000 долларов. Победители финала WSL 2021 года среди мужчин и женщин получат по 200000 долларов. «Для меня бой был в воде», — сказал Мур TIME. «Я пытался доказать, что мы заслужили быть на таком уровне».

После того, как WSL отменил сезон 2020 года из-за пандемии COVID-19, 28-летний Мур вернулся на вершину рейтинга в этом году благодаря четырем сильным выступлениям на мероприятиях в Австралии в апреле и мае. «Лучший способ разогреться перед Олимпиадой, — говорит она, — это убедиться, что все соки от соревнований текут».

В Токио такие соперники, как австралийка Стефани Гилмор, семикратная чемпионка WSL, и 19-летняя американская выскочка Кэролайн Маркс, сразятся с Муром за первое золото в этом виде спорта в Тибе, Япония, на месте олимпийского пляжа для серфинга.

«Я знаю, что мы сделаем все возможное и действительно устроим хорошее шоу», — говорит Мур. «Я надеюсь, что для всех тех маленьких девочек, которые смотрят и мечтают, это действительно вдохновляет их стать большими». — Шон Грегори

Гвен Берри

Легкая атлетика, США

Гвен Берри из США участвует в квалификационных соревнованиях по женскому молотку 27 сентября 2019 года в Дохе, Катар.

Маттиас Хангст — Getty Images

Гвен Берри отказывается заткнуться и бросить молоток. Когда Берри подняла кулак на пьедестале почета во время шоу «Усеянное звездами знамя» на Панамериканских играх 2019 года, Олимпийский и Паралимпийский комитет США назначил ей 12-месячный испытательный срок. Позже она извинилась, но она сказала, что впоследствии потеряла спонсоров и 80% своего дохода.На испытаниях 26 июня, во время которых она прошла квалификацию на свои вторые Олимпийские игры подряд, Берри отвернулась от флага США во время исполнения государственного гимна.

Подробнее

Гимн играл только один раз за ночь, и в тот вечер Берри был на пьедестале почета, заняв третье место в метании молота. «Я чувствую, что это была подстава, и они сделали это специально», — сказал Берри. «Я был зол, если честно.«МОК заявил, что спортсмены могут протестовать на Играх перед соревнованиями, но не на медалях. Берри не отступает. «Когда я доберусь туда, — говорит она, — я придумаю, чем заняться». — Шон Грегори

Джанья Гарнбрет

Спортивное скалолазание, Словения

Джанья Гарнбрет росла, карабкаясь по мебели и взбираясь по деревьям, и с тех пор не останавливается.22-летняя спортсменка широко известна как лучшая женщина в скалолазании и главная надежда ее страны на золотую медаль в Токио.

Подробнее

Она завоевала колоссальные 46 подиумов Кубка мира, а также титулы чемпионов мира по боулдерингу и лазанию на свинью — двух из трех дисциплин, которые составят конкуренцию в олимпийском дебюте этого вида спорта. Третье, скоростное лазание, — ее самое слабое место, но она старалась улучшить его за последний год. С его смелыми прыжками и супергеройскими движениями скалолазание станет настоящим хитом этих Игр, а Гарнбрет — восходящей звездой. Она с радостью взваливает на себя эту ношу. «Я обязан показать этот спорт миру и подать хороший пример», — сказал Гарнбрет TIME ранее в этом году. «Я пойду туда, чтобы насладиться этим, потому что я знаю, что если мне это понравится, все будет в порядке». —Райса Брунер

Масахиро Танака

Бейсбол, Япония

Масахиро Танака из New York Yankees разминается в КПЗ перед игрой против Atlanta Braves 12 августа 2020 года в Нью-Йорке.

Джим МакАйзак — Getty Images

Мечты принимающей страны о бейсболе могут зависеть от первоклассного питчера Масахиро Танака, который вернулся в Тихоокеанскую лигу Японии в этом сезоне после семилетнего пребывания в «Нью-Йорк Янкиз». Танака будет одним из главных звезд Токио, поскольку Высшая лига бейсбола не позволяла активным игрокам соревноваться.

Подробнее

Последний раз бейсбол участвовал в Олимпийских играх в 2008 году, Танака также представлял Японию; эта команда проиграла США в матче за бронзу. «У меня было горькое предчувствие, когда я играл на Олимпийских играх в Пекине, — сказал Танака. «На этот раз я хочу выиграть золотую медаль». Эти Игры будут иметь особое значение для команды хозяев поля, имеющей богатую бейсбольную историю. 28 июля Япония сыграет в Фукусиме, месте ядерной аварии 2011 года, последовавшей за землетрясением и цунами, унесшими жизни около 20 000 человек. — Шон Грегори

даже в бешеной скорости Ледеки на прохождение уходит около 15 минут.

Подробнее

В пятый день соревнований по плаванию в Токио 24-летний спортсмен проплывет финальные дистанции на 200 и 1500 м вольным стилем за одну и ту же двух с половиной часовую сессию. «Я хотела бы отметить, что у мужчин нет двойника», — сказала она. Будет ли это ее беспокоить? Возможно нет. Ледеки, которая пропустила выпускной из Стэнфордского университета из-за того, что участвовала в июньских олимпийских соревнованиях, владеет тремя мировыми рекордами и выиграла пять олимпийских золотых медалей с момента своего дебюта в Лондоне в 2012 году, имеет привычку ставить себя на половину длины тела или больше. впереди своих ближайших соперниц. —Алис Парк

Эйприл Росс

Пляжный волейбол, U.С.

Апрель Росс бьет по мячу во время финала чемпионата AVP Gold Series на Оук-стрит-Бич 1 сентября 2019 года в Чикаго, штат Иллинойс.

Джастин Кастерлайн — Getty Images

Все, что осталось для Эйприл Росс, — золото.Росс выиграла серебряную медаль на Олимпийских играх в Лондоне, играя с Дженнифер Кесси, а в Рио она выиграла бронзу с Керри Уолш Дженнингс, трехкратным золотым призером.

Подробнее

В этом цикле Росс и Уолш Дженнингс разделились; Росс, которому сейчас 39, объединился с Аликс Клайнман, новичком в домашнем волейболе. (Уолш Дженнингс не прошла отбор на свои шестые Олимпийские игры.) Пара провела вместе тренировку по борьбе с пандемией и выиграла свой первый турнир в 2021 году в Дохе, Катар.Росс, которая также играла в волейбол в помещении в колледже — в Университете Южной Калифорнии, где она выиграла два национальных титула — и профессионально, прежде чем сменить кроссовки на песок, говорит, что ее вдохновила продолжить движение ее мать Марджи, которая умерла от рака груди, когда Росс было 19 лет; она каждый день представляет себя на вершине подиума в Токио, ее мама смотрит на нее, улыбаясь. «Это меня волнует, — говорит Росс. «Надеюсь, это движущая сила». — Шон Грегори

Джон Джон Флоренс

Surfing, U.S.

Во время пандемии многие олимпийцы изо всех сил пытались найти места для занятий своим ремеслом. У американского серфера Джона Джона Флоренса, который скрывался на своих родных Гавайях, не было этой проблемы. «Даже во время карантина, комендантского часа и тому подобных вещей океан никогда не закрывался», — рассказывает TIME Флоренс, двукратная чемпионка мира. В преддверии олимпийского дебюта серфинга Флоренс пришлось справиться с травмами: в 2019 году он сломал связку крестообразной связки правого колена, а в мае ему потребовалась операция на левом колене.Но реабилитация прошла хорошо, и Флоренс должна начать заниматься серфингом в Японии, где волны, как правило, менее сильные, чем в его родном штате. «Когда волны действительно маленькие, — говорит он, — вы должны полагаться на свое оборудование, свою физическую форму и все такое, чтобы создать эту энергию и создать такую ​​скорость». — Шон Грегори

Катинка Хоссу

Плавание, Венгрия

Катинка Хоссзу, 32-летняя ветеранка гонок, установила мировые рекорды в индивидуальных комплексных соревнованиях на 200 и 400 м, которые требуют от пловцов владения всеми четырьмя движениями — баттерфляем, баттерфляем. плавание на спине, брасс и вольный стиль — и доминировали в ее соревнованиях в этих соревнованиях на чемпионате мира 2019 года.

Подробнее

Токио станет ее пятой Олимпиадой — и ее первой после разделения с двумя разными тренерами, одним из которых является ее бывший муж. На этот раз Хоссу решила потренироваться. «Как женщина, это иногда немного отличается от [это] для мужчин-спортсменов», — сказала она Sports Illustrated. «Иногда тренеры получают больше уважения, чем спортсмены». Неудивительно, что откровенный венгр, получивший прозвище «Железная леди», получил прозвище «Железная леди» за свою печально известную гонку выносливости в, казалось бы, бесчеловечном количестве соревнований. —Алис Парк

Андре де Грасс

Легкая атлетика, Канада

Канадец Андре де Грасс во время забега на 200 метров среди мужчин на международном стадионе Халифа 29 сентября 2019 года в Дохе, Катар.

Саймон Брути — Sports Illustrated / Getty Images

Он последний выживший. В связи с тем, что Усэйн Болт завершил карьеру, а другие некогда элитные спринтеры не смогли пройти квалификацию, 26-летний Андре де Грасс станет единственным медалистом Игр в Рио-де-Жанейро на дистанциях 100 и 200 метров, участвовавшим в гонках в Токио.

Подробнее

В 2016 году он выиграл бронзу на 100 м и серебро на 200 м, а также бронзу в эстафете 4 × 100 м, став первым канадским спринтером, завоевавшим три медали на одной Олимпиаде.Де Грасс был занят после Рио, окончил Университет Южной Калифорнии и имел двоих детей со своей партнершей Нией Али, чемпионкой мира по бегу с препятствиями из Америки. И в 2017, и в 2018 году сезоны Де Грасса были прерваны из-за травм подколенного сухожилия. Сейчас он здоров, и, поскольку чемпион мира 2019 года на 100 м Кристиан Коулман выбыл из Игр из-за пропуска тестов на наркотики, возможно, де Грассу наконец удалось подняться на подиум. — Шон Грегори

Кевин Санджая Сукамульджо

Бадминтон, Индонезия

Бадминтон — единственный вид спорта, в котором Индонезия выиграла олимпийское золото, и игра здесь так же популярна, как футбол или баскетбол в других частях мира. Кевин Санджая Сукамульджо, лучший в мире парный игрок со своим партнером Маркусом Фернальди Гидеоном, является последней звездой, которая несут надежды его помешанной на бадминтоне нации.

Подробнее

Пара, которую ласково называли «миньонами» из-за их невысокого роста и гиперактивного стиля игры, сохранила свое первое место в рейтинге, несмотря на неудачи в этом году. В январе они были вынуждены отказаться от участия в трех международных турнирах после того, как Сукамульджо заразился COVID-19.В марте команде пришлось отказаться от участия в All England Open после того, как у пассажира их рейса был обнаружен вирус. Это должно быть далеким воспоминанием в Токио, где миньоны готовы добавить к бадминтону славы Индонезии. — Мадлен Роуч

Кристоф Милак

Плавание, Венгрия

Кристоф Милак (Венгрия) участвует в полуфинале мужского забега на 100 м баттерфляем во время соревнований по плаванию на чемпионате мира 2019 года в Муниципальном центре водных видов спорта Университета Намбу 26 июля 2019 года в Кванджу, Южная Корея.

Франсуа-Ксавье Марит — AFP / Getty Images

Кристоф Милак побил мировой рекорд Майкла Фелпса на дистанции 200 метров баттерфляем на чемпионате мира 2019 года, сделав 21-летнего из Венгрии человеком, побившим его на этом мероприятии в Токио.

Подробнее

Милак продолжал набирать обороты в «потерянный» год после перенесения Токийских игр — до осени 2020 года, когда он заразился COVID-19.Милак месяцами боролся с последствиями. Но он оправился, чтобы доказать на чемпионате Европы в мае, что у него есть шанс выиграть первое олимпийское золото на дистанции 200 м баттерфляем в постфелпсскую эпоху. Милак также может пройти квалификацию на дистанцию ​​100 м баттерфляем, где он сможет бросить вызов нынешнему рекордсмену мира Челебу Дресселю. —Алис Парк

Лиза Кэррингтон

Каноэ, Новая Зеландия

Лиза Кэррингтон настолько доминирует в спринтерском каякинге, что ее сравнивают с Симоной Байлз.Она выиграла золото в двух играх подряд (ее победа в 2012 году была первой для маори) и не проигрывала в спринте на 200 метров более десяти лет.

Подробнее

Но она решает свою величайшую задачу в Токио, участвуя в четырех различных соревнованиях. Есть свидетельства того, что ее гамбит может окупиться. На чемпионате мира 2019 года Кэррингтон проиграла соревнования в индивидуальных гонках, а в последний раз, когда она участвовала во всех четырех соревнованиях, в 2018 году она лишь незначительно проиграла командным гонкам.В случае успеха Кэррингтон войдет в историю: еще три золотые медали сделают ее самым титулованным олимпийцем Новой Зеландии за всю историю. —Майкл Зенни

Лорел Хаббард

Тяжелоатлет, Новая Зеландия

Лорел Хаббард из Новой Зеландии во время финальных соревнований по тяжелой атлетике среди женщин в категории свыше 90 кг во время Игр Содружества в Голд-Косте, 9 апреля 2018 года, на Голд-Косте, Австралия.

Дэн Маллан — Getty Images

Лорел Хаббард войдет в историю Токио. 43-летняя штангистка, которая будет выступать за Новую Зеландию в категории женщин свыше 87 кг, станет первой спортсменкой-трансгендером на современных Олимпийских играх — веха, которая привлекла внимание частной спортсменки к ней. не ищу.

Подробнее

Хаббард отвечает всем критериям, установленным МОК для транс-спортсменов с 2015 года, включая ограничения по тестостерону для транс-женщин. Хотя в 2018 году она получила травму, которая могла закончить карьеру, она вернулась к соревнованиям и в настоящее время занимает 15-е место в мире в своем дивизионе. «Мне сообщили, что моя спортивная карьера, вероятно, подошла к концу», — сказал Хаббард в заявлении, когда ее назвали в команду.«Но твоя поддержка, твоя поддержка и твоя ароха несли меня сквозь тьму». —Madeleine Carlisl e

Мария Ласицкене

Легкая атлетика, Олимпийский комитет России

В течение многих лет доминировавшая в прыжках в высоту (выиграв золото на чемпионатах мира 2015, 2017 и 2019 годов), 28-летняя Мария Ласицкене борется за свое первое олимпийское золото. Токио.

Подробнее

Она будет одной из 335 россиян, соревнующихся в Токио под нейтральным флагом после того, как расследование выявило широко распространенную, спонсируемую государством программу допинга спортсменов. В качестве наказания российским спортсменам запрещают соревноваться под своим флагом и гимном до 2022 года. В стране, где ведущие спортсмены редко выступают против официальных лиц, Ласицкене был лидером реформ, призывая к замене высших должностных лиц и тренеров по легкой атлетике.«Я просто спортсменка», — написала она в открытом письме 2019 года. «Но у меня много вопросов». —Madeline Roache

Хелен Гловер

Гребля, Великобритания

MBE Хелен Гловер в действии во время тренировки TeamGB по академической гребле 5 мая 2021 года в Рединге, Англия.

Фото Наоми Бейкер – Getty Images

Если бы Олимпийские игры прошли в 2020 году, как и планировалось, Хелен Гловер там не было бы. Ее близнецы родились в январе того же года, и идея добавить третью золотую медаль к тем, которые она выиграла в Лондоне 2012 и Рио 2016, казалась невозможной. Одна из самых титулованных спортсменок в истории женской академической гребли предполагала, что будет смотреть Токийские игры по телевизору.

Подробнее

Но во время изоляции Гловер начала тренироваться на гребном тренажере и наблюдала, как ее показатели улучшаются, несмотря на то, что она родила троих детей в возрасте до 3 лет. Она серьезно возобновила тренировки и в апреле выиграла титул чемпиона Европы среди женщин в паре вместе со своим партнером по гонкам Полли. Суонн. (Две предыдущие золотые медали Гловера были выиграны у ушедшей на пенсию Хизер Стэннинг.Теперь 35-летний Гловер станет первым британским гребцом, который будет участвовать в Играх после рождения детей. И третье золото вполне достижимо. — Дэн Стюарт

Никита Нагорный

Гимнастика, Олимпийский комитет России

В то время как японские и китайские мужчины доминировали в гимнастике в последних олимпийских циклах, Никита Нагорный находится в авангарде возрождающейся России, стремящейся вернуть себе право хвастаться.

Подробнее

На чемпионате Европы по спортивной гимнастике 2021 года в апреле Нагорный стал первым гимнастом, выполнившим тройное сальто согнувшись назад в своих вольных упражнениях, установив планку для мужского многоборья, которую немногие могут достичь. Популярный в социальных сетях, где он ведет хронику своих тренировок и ежедневных подвигов, в Токио 24-летний игрок не просто присматривается к индивидуальному многоборью, но надеется, что он и его нейтральные товарищи по команде Олимпийского комитета России смогут выиграть первое командное золото для российских спортсменов с 1996 года. —А.П.

Саураб Чаудхари

Стрельба, Индия

Саураб Чаудхари из Индии участвует в финале соревнований по стрельбе из пневматического пистолета на 10 метров среди мужчин на Азиатских играх 2018 года 21 августа 2018 года в Палембанге, Индонезия.

Эдгар Су — Рейтер / Алами

Стрелок Саураб Чаудхари, 19 лет, является одним из лучших стрелков Индии в Токио. Занявшись стрельбой только в 2015 году, он накопил внушительную сумму медалей: 14 золотых и шесть серебряных на международных соревнованиях. В 2018 году он стал самым молодым золотым медалистом Индии на Азиатских играх.

Подробнее

Год спустя Чаудхари установил новый мировой рекорд среди юниоров и взрослых в категории 10-метровых пневматических пистолетов на чемпионате мира Международной федерации стрелкового спорта, обеспечив себе место в сборной Индии по стрельбе из 15 человек в Токио. Там он также сыграет в паре с Ману Бхакером, чтобы стать лучшим соперником в смешанном командном соревновании. «Он чрезвычайно сосредоточен, — сказал товарищ по команде Апурви Чандела. «Я вижу, что в будущем с ним произойдут великие дела. — Мэделин Роуч

Ноа Лайлс

Легкая атлетика, США

Ной Лайлс с 2016 года пробежал четыре самых быстрых 200-метровых раза в мире, но вы бы не знали об этом на старте олимпийских испытаний в июне. . После того, как 23-летний игрок из Флориды не смог выиграть свой забег или полуфинал, некоторые задались вопросом о его форме и о том, была ли шумиха по поводу того, что он может стать следующим Усэйном Болтом, преувеличена.

Подробнее

Но в финале он кричал во главе стаи, чтобы попасть в свою первую олимпийскую команду и зарекомендовать себя как фаворит на победу в беге на 200 м — чего ни один американец не делал с 2004 года. Лайлс открыто рассказывал о своей борьбе. с психическим здоровьем и давлением конкуренции. Прошлым летом он поделился, что начал принимать антидепрессанты, и назвал это «одним из лучших решений, которые я принял за последнее время.«Отправляясь в Токио, он говорит, что нашел ясность. «Мне нечего доказывать, — говорит Лайлс. «Когда я проведу гонку, у вас будут проблемы». — Шон Грегори.

Элиуд Кипчоге

Марафон, Кения

Элиуд Кипчоге из Кении, обладатель золотой медали, пересекает финишную черту и празднует с флагом во время марафона NN Mission Marathon 18 апреля 2021 года в Энсхеде, Нидерланды.

Дин Мухтаропулос — Getty Images

«Лично я не верю в ограничения», — сказал однажды Элиуд Кипчоге, кенийский марафонец. Как единственный человек в истории, который пробежал марафон на 26,2 мили менее чем за два часа, почему он должен?

Подробнее

Этот подвиг 2019 года в Вене может не считаться официальным мировым рекордом, поскольку Кипчоге не участвовал в гонках.Соревнование было спроектировано для максимальной скорости: трасса была ровной, а команда кардиостимуляторов окружила Кипчоге, чтобы уменьшить сопротивление. Но Кипчоге держит мировой рекорд: он пробежал его за 2 часа. 1 мин. 39 сек. в Берлине в 2018 году. В апреле он выиграл свой последний марафон — в аэропорту Нидерландов — перед Олимпиадой, где Кипчоге будет защищать золото, завоеванное им в Рио. На это у него большие деньги. — Шон Грегори

Shaunae Miller-Uibo

Легкая атлетика, Багамы

Shaunae Miller-Uibo — действующий олимпийский чемпион в беге на 400 м, выиграв золото в Рио-де-Жанейро, прыгнув через финишную черту. вытеснить Эллисон Феликс.Драматический момент стал чем-то вроде теста Роршаха: нарушил ли Миллер-Уйбо неписаные правила спортивного поведения или просто сделал все, что нужно для легальной победы?

Подробнее

27-летняя спринтерка, несшая флаг своей страны в 2016 году, неудивительно, что она придерживается второй точки зрения. «У меня было много людей, которые приходили ко мне и говорили:« После того, как мы увидели, что вы делаете … это заставило меня погрузиться в мои мечты и еще больше усердно их преследовать », — недавно сказал Миллер-Уйбо в интервью Olympic Channel. Миллер-Уйбо могла пробежать как 200 м, так и 400 м в Токио, но она сказала, что сосредоточится на 200 м, в которых она является рекордсменом Багамских островов, поскольку расписание затрудняет преодоление двойного удара. «Да, было бы здорово защитить свой титул», — говорит Миллер-Уйбо. «Но в то же время я хотел новое название». — Шон Грегори

Небесно-коричневый

Скейтбординг, Великобритания

Sky Brown в скейтпарке Vans Off The Wall Хантингтон-Бич в 2020 году в Лос-Анджелесе, Калифорния.

Хранитель / Eyevine / Redux

Скай Браун уже заработала контракт с Nike, появилась на Танцы со звездами: юниоры , записала поп-песню и изображает куклу Барби, а 7 июля ей исполнилось 13 лет. Великобритания, мать которой из Японии, будет соревноваться за сборную Великобритании по скейтбордингу в парке в Токио благодаря раннему арсеналу движений, включая 720 — два полных оборота в воздухе.

Подробнее

Ее навыки вдохновляли фигуристок всех возрастов. «Она чертовски крутая», — говорит 18-летняя Джоселин Писатель, которая катается на коньках в Венеции, Калифорния, где иногда тренируется Браун. «Видеть, как маленькая девочка становится лучше, чем, например, половина фигуристов, это очень воодушевляет». Игры проходят чуть больше года с тех пор, как Браун получила перелом черепа и сломала левое запястье и руку в ужасающем падении во время катания на коньках.»Ничего страшного. иногда падать », — сказала она в сообщении на YouTube со своей больничной койки. «Я просто собираюсь встать и надавить еще сильнее». — Шон Грегори

Юто Хоригоме

Скейтбординг, Япония

Юто Хоригоме написал в своем ежегоднике начальной школы, что его мечтой было «стать лучшим скейтбордистом в мире». Уроженец Токио начал заниматься скейтбордингом в возрасте 6 лет, сопровождая своего отца в скейт-парки.Сейчас ему 22 года и он занимает второе место в мире по уличному катанию среди мужчин. Он может достичь этой цели эффектным способом, когда этот вид спорта дебютирует на Олимпийских играх в его родной стране.

Подробнее

Чтобы добраться до этого места, Хоригоме пришлось покинуть дом. Он начал ездить в США, центр соревновательного катания на коньках, на соревнования еще подростком в 2014 году.Всего четыре года спустя Хоригоме стал первым японским фигуристом, завоевавшим титул чемпиона мира в рамках тура Street League Skateboarding в Лондоне. После окончания средней школы Хоригом переехал в США и сейчас живет в Лос-Анджелесе, где купил дом с собственным скейт-парком.

Переезд Хоригома через океан был продиктован как амбициями, так и необходимостью. Фигуристы в Японии давно считаются нарушителями спокойствия. На городских улицах нет знаков скейтбординга, и фигуристы говорят, что охранники или полиция преследуют их даже за то, что они носят скейтборды с собой.Но растущая популярность этого вида спорта помогла ослабить стигму, открыв путь для большего количества скейт-парков и мест, которые помогают развивать культуру катания на коньках в стране. А поскольку этот вид спорта находится на пороге Олимпиады, японские СМИ усилили освещение соревнований, и теперь лучшие фигуристы появляются на обложках журналов и в телеинтервью.

Хоригоме — фигурист-новатор, известный тем, что придумывает трюки, которые никто другой не делал (например, фейки с переключающимся задом 180 с гриндом) и совершает сложные вращения и скольжения на соревнованиях, — это лучшая надежда Японии на золото в мужском катании.Для этого ему нужно будет встретиться с Найджей Хьюстон, лучшим уличным фигуристом в мире. Но Хоригоме делал это раньше; В июне он обыграл Хьюстона на чемпионате мира, лишив американца четвертого подряд титула.

Хоригоме сказал репортерам, что победа вселила в него уверенность в том, что он может взять первое золото в фигурном катании в Токио: «Я хочу добиться того, чего еще никто не делал». —Ария Чен

Саид Моллаей

Дзюдо, Монголия

Саид Моллаеи на чемпионате мира по дзюдо 28 августа 2019 года в Токио, Япония.

Наоки Нисимура — Афло Спорт / Алами

Саид Моллаи возвращается в Токио, чтобы избавиться от болезненных воспоминаний. В 2019 году дзюдоисту из Ирана было приказано официальными лицами Тегерана провести полуфинал чемпионата мира по дзюдо в японской столице, чтобы ему не приходилось соревноваться в финале с соперником из Израиля, который Иран не признает государством.

Подробнее

Моллаи, бывший в то время золотым призером соревнований в весовой категории до 81 кг, отказался, но все равно проиграл бой, что он объяснил эмоциональным напряжением. Опасаясь возмездия за свое непослушание, Моллаи с тех пор не вернулся в Иран и вместо этого участвует в Олимпийских играх 2020 года за Монголию, которая предоставила ему гражданство. Но 29-летний футболист тренируется к Играм вместе с израильской командой — теперь это его главные соперники и лучшие друзья.Их доброта, сказал он Международной федерации дзюдо, «я никогда не забуду». — Чарли Кэмпбелл

Рейс Имбоден

Фехтование, США

После того, как Рейс Имбоден встал на колени на медальном стендах Панамериканских игр 2019 года в Перу, сославшись на расизм, насилие с применением огнестрельного оружия, жестокое обращение с иммигрантами и риторику президента Трампа в качестве причин для своего протест — Олимпийский и Паралимпийский комитет США (USOPC) назначил ему 12-месячный испытательный срок, что он назвал «трусливым изгибом».

Подробнее

Но Имбоден, двукратный олимпиец, входящий в состав Совета сборной США по расовой и социальной справедливости, помог вдохновить на перемены: с тех пор ОПК США разрешил протесты на олимпийских испытаниях, заявив, что не будет наказывать спортсменов, мирно демонстрирующих на Играх. В свободное от тренировок время ди-джеем и манекенщицей Имбоден выиграла командную бронзу на рапирах в Рио и командное золото на чемпионате мира 2019 года. — Шон Грегори

Lilly King

Плавание, США

Лилли Кинг из США после участия в полуфинальном заплыве на дистанции 200 метров брассом среди женщин на соревнованиях олимпийской сборной по плаванию 17 июня 2021 года в Омахе, штат Небраска.

Мэдди Мейер — Getty Images

Лилли Кинг увезла домой две золотые медали в Рио, но ее соперничество с российской пловчихой Юлией Ефимовой, которая провалила допинг-тест перед соревнованиями 2016 года, действительно покорило зрителей; Кинг в конечном итоге превзошла ее в финале на дистанции 100 м брассом после вирусного виляния пальцами, и они остаются заметно недружелюбными.

Подробнее

Добраться до Токио для 24-летнего Кинга было другим делом. Когда в начале этого года скончался ее близкий друг и партнер по тренировкам Энни Лазор, Кинг пообещала маме своего товарища по команде, что сделает все возможное, чтобы Лазор стал частью команды. И она сделала; Лазор выиграл 200-метровый брасс на Trials, а Кинг занял второе место. «Это действительно особенный момент между нами двумя, — сказал Кинг.«Мы прошли через ад и снова вместе». —Райса Брунер

Шелли-Энн Фрейзер-Прайс

Легкая атлетика, Ямайка

До дальнейшего уведомления 34-летняя Шелли-Энн Фрейзер-Прайс — самая быстрая женщина в мире. 5 июня двукратный олимпийский чемпион в беге на 100 м пробежал второй рекорд в истории — 10,63 секунды.

Подробнее

Только Флоренс Гриффит Джойнер, установившая мировой рекорд с 10.49 в 1988 году бегала быстрее. В 2017 году, после того как Фрейзер-Прайс выиграла бронзу на дистанции 100 м в Рио, она родила сына Зайона (который заработал ей подходящее прозвище: Mommy Rocket). Она вернулась на трассу, чтобы выиграть золото на чемпионате мира в 2019 году, и теперь она стала женщиной, побежденной в масштабном спринте в Токио.

Победа сделает ее первой женщиной, выигравшей три 100-метровых золотых медали, и самой старой женщиной, совершившей этот подвиг.

Направляясь на свои четвертые Олимпийские игры, Фрейзер-Прайс — самая большая звезда ямайской команды, которая всегда возлагает большие надежды на трассу.В спорте, который, как правило, не поощряет долголетие, Mommy Rocket остается твердой ставкой на успех. — Шон Грегори

Брук Рабуту

Спортивное скалолазание, США

Брук Рабуту из США принимает участие в финале Кубка мира по скалолазанию IFSC в Солт-Лейк-Сити на Industry SLC 22 мая 2021 года в Солт-Лейк-Сити, штат Юта.

Энди Бао — Getty Images

Технически Брук Рабуту — студентка колледжа, специализирующаяся на маркетинге в Университете Сан-Диего. Но этим летом у 20-летней девушки есть медали.

Подробнее

Поскольку спортивное скалолазание дебютирует на Олимпийских играх в Токио, то же самое и с Рабуту; и она первая американка, прошедшая квалификацию на Игры в этом виде спорта.Дочь профессиональных скалолазов и чемпионов мира по скалолазанию Робин Эрбесфилд-Рабуту и ​​Дидье Рабуту, она росла, увлеченная этим видом спорта в Колорадо, и с детства тренировалась, часто вместе с легендами. Она оставила свой след в спорте в молодом возрасте; когда в 11 лет она стала самым молодым человеком, когда-либо пролезавшим маршрут с уровнем сложности 5. 14b по десятичной системе Йосемити. (Маршруты восхождения начинаются с 5.0 и доходят до 5.15d в этой системе). Совсем недавно она заняла первое место на Молодежном Панамериканском чемпионате 2017 года в комбинированном виде, включающем все три дисциплины: скорость, боулдеринг и лазание на свинец, и завоевала свое олимпийское место, заняв девятое место в комбинированном финише на олимпийских квалификациях в Японии. .Учитывая ее молодость, Raboutout можно будет смотреть не только в Токио, но и на долгие годы. —Райса Брунер

Сидней Маклафлин

Легкая атлетика, США

Будущее легкой атлетики США должно прибыть в Токио. В Рио Сидней Маклафлин, которому тогда было 16 лет, стал самым молодым легкоатлетом США, прошедшим квалификацию на Олимпиаду с 1980 года.

Подробнее

Хотя она и не вышла в финал на своих первых Олимпийских играх, с тех пор она догоняет золотую медалистку в беге на 400 метров с барьерами 2016 года Далилу Мухаммед.

На чемпионате мира 2019 года в Дохе, Катар, Маклафлин финишировал вторым после своего коллеги-американца Мухаммеда, который установил мировой рекорд — 52,16 секунды, чтобы выиграть. На соревнованиях по легкой атлетике в США в июне этого года Маклафлин побил этот рекорд, пробежав рекордную 51-ю.9 сек. Ожидайте, что выпадет больше рекордов, когда Маклафлин и Мухаммед встретятся в Токио. Поединок должен быть назначен на просмотр: финал состоится 4 августа — Шон Грегори

Shi Tingmao

Дайвинг, Китай

Ши Тинмао из Чунцина участвует в финале женского 3-метрового трамплина на Национальном чемпионате Китая по прыжкам в воду 2021 года и Олимпийских испытаниях в Токио 15 мая 2021 года в Шанхае, Китай.

VCG / Getty Images

Ши Тинмао так часто побеждала в прыжках с трамплина, что государственные СМИ Китая прозвали ее «генералом-победителем». Действительно, с 2015 года она выиграла все крупные международные соревнования, кроме пяти, в том числе две золотые медали на Играх в Рио в 2016 году.

Подробнее

Ши начала заниматься гимнастикой еще юной девушкой и не присоединилась к национальной сборной Китая по прыжкам в воду, пока ей не исполнился 21 год.Токио будет вторыми Олимпийскими играми Ши, но, вероятно, последними, и она полна решимости закончить их на высокой ноте. «Упорство, самодисциплина, сосредоточенность, финальный спринт, давай!» Ши написал в социальной сети Weibo, похожей на Twitter, накануне Игр. Эти слова совпадают с ее именем, Тинмао, что переводится как «усердно трудиться для страны».

Ши, похоже, продолжит доминировать и поможет сохранить олимпийскую победную серию Китая в индивидуальных прыжках с трамплина среди женщин, которая началась с 1988 года. —Ария Чен

Тедди Ринер

Дзюдо, Франция

«Дайте Тедди Ринеру, одному из самых выдающихся спортсменов спорта, всемирную олимпийскую славу, которую он заслуживает», — гласит недавний заголовок в Eurosport. В самом деле, Ринер управлял дзюдо так же, как немногие спортсмены в любом виде спорта.

Подробнее

Прежде чем проиграть матч в феврале 2020 года, прямо перед закрытием пандемии, Ринер не проигрывал почти десять лет, выиграв 154 боя подряд, одновременно завоевав две олимпийские золотые медали и шесть чемпионатов мира.6 футов. 8 дюймов, 300 плюс фунт. дзюдоист, известный как Большой Тед, вернулся на вершину пьедестала почета на World Masters в Дохе, Катар, в январе. Олимпийское золото в Токио станет для него третьим подряд. — Шон Грегори

Вильфредо Леон

Волейбол, Польша

Вильфредо Леон Венеро из Польши набирает очко против Франции во время Евроволлея 28 сентября 2019 года в Париже, Франция.

Катрин Стинкесте — Getty Images

Польша никогда не была очагом волейбола. Но это было до того, как Вильфредо Леон, человек, известный как Криштиану Роналду, прибыл в страну и в одиночку превратил ее в соперника.

Подробнее

6 футов.8-дюйм. Внешний нападающий дебютировал за сборную своей родной Кубы в 14 лет, а к 17 был ее капитаном — самым молодым в истории. После бегства с Кубы Леон получил польское гражданство в 2015 году. Он играл за российские и итальянские профессиональные команды, став одним из самых высокооплачиваемых волейболистов в мире, но он не получил права выступать за сборную Польши до 2019 года. Куба «будет» всегда есть место в моем сердце », — сказал Леон, но« все, что я делаю в этот момент », предназначено для Польши. С Леоном в составе Польша превратилась в претендента на медали с шансом выиграть свое первое олимпийское золото в этом виде спорта с 1976 года. —Madeline Roache

Юкико Уэно

Софтбол, Япония

После того, как софтбол был исключен из Игр 2012 и 2016 годов, Япония настояла на его возвращении в Токио. Неудивительно, почему такой кувшин, как Юкико Уэно, ждет своего часа. Ветеран-правша бросает одну из самых быстрых веревок в мире — до 80 миль в час — и имеет историю крупных побед на Олимпийских играх.

Подробнее

В Афинах в 2004 году Уэно провел идеальную игру за семь иннингов — единственную в истории Олимпийских игр. На Играх в Пекине в 2008 году она бросила 413 веревок за два дня, в то время как Япония выиграла золото. Ее выступление произвело такую ​​фурор, что 413 передач Уэно были названы одним из главных модных словечек года в Японии. В 38 лет у нее есть еще один шанс снова продемонстрировать свое исключительное доминирование на мировой арене — на этот раз на родине. —Ария Чен

Лаура Кенни

Велоспорт на треке, Великобритания

Британка Лаура Кенни участвует в женской гонке Omnium Tempo Race во время чемпионата мира по трековому велоспорту UCI 28 февраля 2020 года в Берлине, Германия.

Одд Андресен — AFP / Getty Images

Когда в феврале 2020 года она сломала руку в результате аварии, Лаура Кенни была готова полностью отказаться от велотрека. Но перенос Игр в Токио оказался замаскированным благом. Кенни полностью выздоровел, и у него есть дополнительный год, и он стал фаворитом соревнований по велоспорту в закрытых помещениях.

Подробнее

Кенни мгновенно стала британской иконой на Олимпийских играх в Лондоне, когда в возрасте 20 лет она выиграла две золотые медали на глазах у местных жителей. Четыре года спустя она удвоила свой улов в Рио. Токио станет ее первой Олимпиадой с тех пор, как она родила сына Альби в 2017 году. Хотя 29-летняя женщина уже имеет больше золотых медалей, чем любая другая британская женщина в олимпийской истории, у нее есть шанс выиграть еще три в Токио. — приближая ее к тому, чтобы стать самой успешной олимпийской женщиной всех времен и народов из любой страны. — Билли Перриго

Симона Мануэль

Плавание, США

Когда Симона Мануэль взглянула на свое время в полуфинале на дистанции 100 м вольным стилем на олимпийских испытаниях 17 июня, она не была полностью удивлена, даже если большая часть мира плавания был. Она финишировала девятой, пропустив выход в финал и убедившись, что действующий золотой призер на дистанции 100 м вольным стилем не защитит свой олимпийский титул.

Мануэль рассказала после неутешительного заплыва, что в марте у нее диагностировали синдром перетренированности. «Было действительно трудно [плавать] в определенное время, которое было легче за несколько недель или месяцев до этого», — рассказывает она TIME. «Поскольку я продолжал соревноваться и упорно тренироваться, становилось все хуже и хуже».

Подробнее

Ее первым симптомом было учащенное сердцебиение, даже когда она отдыхала или выполняла простые подходы на практике.Когда дела в бассейне не улучшались, это переросло в беспокойство, бессонницу и депрессию.

Ее врач рекомендовал полный отдых, поэтому в марте Мануэль вернулся домой в Техас примерно на 12 дней. «Когда я вернулась домой, моя работа заключалась в том, чтобы буквально выздоравливать», — говорит она. Продолжительный массаж, ледяные ванны и время без воды помогли восстановить ее тело, но ее разум был другим делом. Вынужденный брать так много свободного времени так близко к олимпийским соревнованиям не помогал ее тревоге. «Быть ​​вне воды и смотреть, как время проходит мимо меня, было тяжело психологически», — говорит она.

Это был последний сбой в изнурительном году, когда Олимпийские игры были отложены из-за пандемии COVID-19, которая непропорционально затронула цветных людей, и повсеместного расплаты за расизм. «То, с чем чернокожему сообществу пришлось столкнуться в этом году, было очень трудным, — говорит Мануэль, которая стала первой афроамериканкой, завоевавшей олимпийское золото в плавании в Рио в 2016 году. Ахмауд Арбери и инцидент с мужчиной в Центральном парке, произошедший сразу после друг друга, действительно заставили меня сесть, и я думаю, огорчил некоторые переживания, которые у меня были в спорте, или переживания, которые я получил как чернокожая женщина, которая Я просто отмахнулся раньше.«Это было, по ее словам,« как зеркало, от которого я не могла оторваться ».

Однако время на размышления оказалось потраченным не зря. Через три дня после пропуска финала на 100 м Мануэль выиграла финал на дистанции 50 м вольным стилем на испытаниях, что дало ей шанс заплыть за индивидуальную медаль в Токио.

«Были моменты, когда я не была уверена, смогу ли я попасть в команду», — говорит она. «Но я узнал, что во мне много упорства, много борьбы. Я завелся к этой стене.Больше всего на свете [опыт испытаний] научил меня, что я действительно ничего не могу сделать. И я думаю, что это довольно круто ». —Алис Парк

Liz Cambage

Баскетбол, Австралия

Лиз Камбедж из команды Las Vegas Aces в борьбе с вашингтонскими мистиками 5 июня 2021 года в Вашингтоне, округ Колумбия.

Скотт Тэтч — Getty Images

Некоторое время казалось, что женская сборная Австралии по баскетболу может отправиться на Олимпиаду без одной из своих главных звезд. В мае Лиз Камбедж объявила в социальных сетях, что не будет участвовать в Олимпийских играх из-за обвинений в том, что Олимпийский комитет Австралии обелил маркетинговые материалы.

Подробнее

На фотографии, размещенной на официальной странице австралийской олимпийской сборной в Instagram, изображена форма команды, на которой изображен только один спортсмен из числа коренного населения. Кембедж, чей отец — нигериец, также не согласился с фотографией — в которой, по ее словам, не было расового разнообразия — для бренда Jockey, который поставляет нижнее белье олимпийским спортсменам Австралии. «Если я сказал это однажды, я сказал это миллион раз, — написал 29-летний мужчина в социальных сетях, — как я должен представлять страну, которая даже не представляет меня?» В конце концов, Кембадж, который играет за «Лас-Вегас Эйсес» WNBA, но родом из Мельбурна, передумал.«Я собираюсь бросить вызов всем тем молодым смуглым детям в Австралии, которые смотрят на меня, детка», — сказала она. «Я сделаю это за тебя. — Эми Гуния

Суниса Ли

Гимнастика, США

Ни одна гимнастка не имеет более сложного упражнения на брусьях, но вы не узнаете его по тому, как Суниса Ли выполняет его. Спокойный, элегантный поток 18-летней жительницы Санкт-Петербурга.Пол, Миннесота, обладает внутренней силой, благодаря которой самые опасные навыки кажутся легкими.

Подробнее

Сила духа Ли была испытана другим способом в 2019 году, когда за несколько дней до ее отъезда на национальный чемпионат США ее отец упал, помогая другу подрезать дерево; он остается частично парализованным. Он перенес операцию, пока Ли соревновалась, но ей все же удалось заработать серебро за Симоной Байлз.Эта сила будет проверена снова, когда она будет участвовать в своих первых Олимпийских играх — как первая американка хмонг, попавшая в сборную США — снова отделенная от своей дружной семьи, на этот раз из-за пандемии. —Алис Парк

Дина Ашер-Смит

Легкая атлетика, Великобритания

Самая быстрая британка в истории ехала драматическим маршрутом в Токио после того, как неисправность часов на национальных олимпийских соревнованиях создала путаницу в отношении победы спринтера на 100 метров.Хотя она и не побила свой национальный рекорд, как она — и публика — изначально думала, ее спринта в 10,97 секунды было достаточно, чтобы обеспечить себе место в верхней части легкоатлетической команды Team GB.

Подробнее

25-летняя спортсменка бьет рекорды и завоевывает медали с 13 лет, в том числе бронзу в эстафете 4х100 м на Олимпийских играх в Рио. Она чемпионка мира в беге на 200 м и самая большая надежда Великобритании на золото на беговой дорожке в Токио. И если она получит медаль в беге на 100 метров — возможность, предоставленная ей в последнее время, — она ​​станет первой британкой, которая сделает это за более чем 60 лет. — Аеша Джаве д

Подробнее об Олимпийских играх в Токио:

Еще истории, которые необходимо прочитать из TIME


Свяжитесь с нами в письмах @ время.com.

Новые исследования в области науки, технологий и математики (NEST + m) — Район 1

ОБЗОР 2018 г. : одна из пяти общегородских программ для одаренных детей и единственная программа, предназначенная для обслуживания детей в классах K-12, «Новые исследования в науке и технологиях» В программе & Math (NEST + m) сложная учебная программа, чрезмерно вовлеченные родители и дети, которые любят ходить в школу.

Новые ученики принимаются в детский сад, в 6-й и 9-й классы, а некоторые принимаются в другие классы.Хотя некоторые учащиеся остаются здесь на все 13 лет, многие уезжают в крупные специализированные средние школы и другие конкурентоспособные школы, поэтому имеется значительное количество мест для новых учеников 9-х классов. В то время как большинство детей младших классов — белые и азиатские, в старшие классы поступает больше чернокожих и латиноамериканцев.

В начальных классах сочетаются прогрессивные и традиционные методы. В детских садах упор делается на академические навыки, а не на драматические игры: в них нет уголков для переодевания или игровых кухонь, как, например, в более прогрессивных школах.Сверхактивный PTA платит помощникам учителей в младших классах.

NEST + m использует сингапурскую математику в классах K-5, динамичный и продуманный учебный план, сочетающий быстрое вспоминание арифметических фактов с глубоким пониманием математических концепций. Около 50 детей 2-8 классов принимают участие в математической команде, которая собирается в 7 утра несколько раз в неделю; эти дети участвуют в соревнованиях с другими школами.

В уроки социальных наук сплетены вокруг междисциплинарных тем, таких как Центральный парк (как читать карты или строить мосты, или какие дикие животные там обитают).Школа принимает участие в проекте Педагогического колледжа по чтению и письму, в котором дети пишут несколько черновиков сочинений, прежде чем «опубликовать» окончательный вариант.

Марк Берковиц, который стал директором школы в июле 2015 года, внес значительные изменения, особенно в средних и старших классах школы. Он заменил ряды парты маленькими столиками, чтобы стимулировать обсуждения в классе и групповую работу, а не лекции учителей. Он расширил списки чтения, включив в них больше чернокожих и латиноамериканских авторов.Он работал с персоналом над развитием образования, учитывающего культурные особенности, то есть методов обучения, которые лучше отражают опыт небелых детей.

Осенью 2018 года африканский писатель и изобретатель Уильям Камквамба посетил школу, чтобы рассказать о своей книге Мальчик, запряженной ветром , рассказе о своем опыте создания ветряной мельницы для подачи электричества в свою деревню в Малави. Берковиц сказал, что эта книга и визит являются примерами того образования, которое он хочет развивать; работа, основанная на разных дисциплинах, таких как литература и наука, и знакомит детей с разными культурами.

Возможно, самый большой сдвиг в философии школы — это подход к учащимся с ограниченными возможностями. В средних и старших классах школы Берковиц добавил классы группового обучения с двумя учителями, один из которых имеет сертификат специального образования, которые смешивают учащихся с ограниченными возможностями и учащихся общеобразовательных школ.

Хотя для зачисления учащиеся из обеих групп должны быть хорошо успевающими, стандарты для людей с ограниченными возможностями немного более гибкие. (Смена не коснулась начальной школы, в которой услуги специального образования ограничены. Прием в начальную школу по-прежнему основан на сдаче городского экзамена для одаренных.)

Реакция на изменения неоднозначная. Одна мать сказала, что это «более теплое и счастливое место». Она сказала, что директор знает каждого ребенка по имени и, кажется, любит разговаривать со студентами. Другой сказал, что «дети серьезно относятся к своей работе», но учителям «удается все сделать весело». Другие родители хвалят учителей и офис колледжа.

Но некоторые родители и учителя критикуют стиль управления Берковица и говорят, что он не реагирует на их опасения.Согласно опросам школ, значительное число учителей не доверяют директору школы. Некоторые утверждают, что изменения произошли за счет академической строгости и что новый акцент на английском языке и истории произошел за счет математики и естественных наук.

Берковиц отвечает, что стандарты как никогда высоки, указывая на надежные предложения по продвинутому трудоустройству и прием в очень избирательные колледжи и университеты, включая MIT, Wesleyan, Swarthmore, Yale, Columbia, Brown and Cooper Union. Ранее он был заместителем директора в NYC Lab School for Collaborative Studies, школе, которая гордится тем, что обслуживает успешных учеников с ограниченными возможностями.

СПЕЦИАЛЬНОЕ ОБРАЗОВАНИЕ : В классах начальной школы дети с ограниченными возможностями, такими как дислексия и СДВГ, получают SETSS (услуги по поддержке учителей специального образования). Детям помогают профессиональные, физиотерапевты и логопеды, а психолог работает два дня в неделю. В средних и старших классах школы есть классы ИКТ или группового обучения для учащихся с различными формами инвалидности.

ЗАПИСЬ : Воспитанники детского сада должны набрать 99-й процентиль на городском экзамене на одаренных и талантливых, чтобы их даже рассматривали. (Братья и сестры нынешних учеников могут подать заявку, если они набрали 97-й процентиль.)

При зачислении в детский сад родители должны подать заявку на тестирование в октябре; дети проходят тестирование в январе.

Учащиеся средних и старших классов принимаются на основании их оценок, посещаемости и результатов государственных экзаменов. Учащиеся общеобразовательных школ, как правило, должны иметь средний балл 90 или выше и набрать не менее 3 баллов по стандартным тестам; учащиеся с ограниченными возможностями, которым требуются групповые занятия, могут иметь средний балл 85 и тестовые баллы 2.75.

Когда школа добавляет класс, в 6-м классе появляется около 30 новых мест; в 7-м классе появляется 10-15 новых мест, когда некоторые ученики уезжают в среднюю школу Хантер-Колледжа. В 9 классе более 100 новых мест.

Текущим ученикам начальной школы гарантирован прием в среднюю школу; нынешним учащимся средней школы гарантируется зачисление в старшую школу. Бесплатный желтый школьный автобус предоставляется для детей классов K-6, живущих на Манхэттене, в 5,5 милях от школы.Родители в других районах и других частях Манхэттена могут оплачивать услуги частного автобуса. (Clara Hemphill, сентябрь 2018 г. )

Задания для олимпиад на английском языке. Бесплатные интернет-олимпиады школьников по английскому языку

Английская олимпиада

школьный этап

2015-2016 учебный год

Пояснительная записка

Основными целями и задачами олимпиады являются выявление и развитие у

учащихся творческих способностей и интереса к английскому языку, создание необходимых условий для поддержки одаренных детей, продвижение языковых и социокультурных знаний, связанных с историей и современным функционированием различных версии английского языка, истории и культуры англоязычных стран.

Задания олимпиады носят проблемно-поисковый характер и раскрывают

творческого потенциала участника, соответствуют всем тестовым требованиям

заданий.

Задача школьного этапа олимпиады по английскому языку — популяризация английского языка в школе, привлечение как можно большего количества учеников к участию в олимпиаде, поэтому задания должны быть интересными и посильными для учеников соответствующих возрастных групп. .

Для комплексной проверки уровня коммуникативности

компетенции участников школьный этап олимпиады проводится по трем видам речевой деятельности:

, конкурс на понимание прочитанного (чтение)

лексико-грамматический тест ( Использование английского языка)

письменный конкурс (Writing).

для 5-6 классов — А1-А2;

для 7-8 классов — А2-В1;

для 9-11 классов — В1 -В2.

При подготовке заданий комбинируются задания разных уровней сложности (т.е.

более сложные и менее сложные задания объединяются, чтобы участники могли выполнить хотя бы одно олимпиадное задание). Уровни сложности разных заданий в пакете заданий для одной возрастной группы не отличаются более чем на один шаг.

Время проведения соревнований:

5-6 классы — 50 минут

7-8 классы — 50 минут

9-11 классы — 75 минут

Максимальное количество баллов

5-6 классы -85 баллов

7-8 классы — 30 баллов

9-11 классы — 50 баллов

Комплект заданий для учащихся 5-6 классов

Чтение

15 минут.

Задача 1.

Прочтите текст о Мартине и отметьте предложения буквой (T), если информация верна, и буквой (F), если она неверна.

Моя одежда.
Мне нравятся брюки и джинсы. Не люблю рубашки, но люблю футболки и свитшоты. Еще мне нравятся бейсболки. Мои любимые цвета — синий и красный.
В школе я обычно ношу черные брюки или синие джинсы, футболку или толстовку. У меня много футболок.Моя любимая футболка синего цвета с изображением тигра. Я обычно ношу кроссовки. У меня есть черные туфли, но они мне не нравятся.
Сейчас на мне синие джинсы, красно-синяя толстовка и мои новые кроссовки. Мне очень нравятся мои новые кроссовки!

1. Мартин любит джинсы. ___
2 … Не любит толстовки. ___
3 … Его любимые цвета — синий и зеленый. ___
4 … В школе он обычно носит зеленые брюки.___
5 … Его любимая футболка черная. ___
6 … Ему не нравятся его черные туфли. ___
7 … Мартин носит кроссовки. ___
8 … В данный момент на нем зелено-синяя толстовка. ___
9 … Он не любит кроссовки. ___
10 … У Мартина много футболок .___

Задача 2

Прочтите текст и введите правильное слово.

л ове парк мясной хвостик большой

Моего пса зовут Сэм и ему три года.Это черный и _________ (11) и у него длинный ___________ (12) … У него тоже большие уши.

Я должен брать Сэма на прогулку каждый день. Беру в _______________ (13) с папой. Сэм может быстро бегать, но не умеет ловить мячи. Сэм должен съесть много _____________ (14) , но нельзя есть мороженое. Он также должен пить много воды. Он спит в моей спальне.

И _______________ (15) Сам, потому что он дружелюбный и умный.Когда мне грустно, Сэм снова делает меня счастливым!

Использование английского языка

15 минут.

Задача 1.

Прочтите письмо английского мальчика Джона и выберите подходящие слова.

Дорогой друг по переписке,

Здравствуйте! Мое имя (16) Иоанна. Я (17) в Бостоне. Я (18) получил большой дом. Есть (19) четыре комнаты в доме. Есть (20) гостиная, столовая и две спальни в доме.Наша кухня очень большая и мама (21) там много вкусного. Я (22) моя комната с моим братом Тедом.

Следующим летом ср (23) в лагерь вместе, и мы там повеселимся. Мой (24) продукты питания рыбные. Мой отец работал в банке. Он (25) , что в Австралии много людей из Бостона.

Напишите нам в ближайшее время.

Пока,

Джон.

16

1) это

2) am

3) это

17

1) живет

2) живет

3) живет

18

1) будет

2) будет

3) будет

19

1) это

2) am

3) это

20

1) это

2) am

3) это

21

1) готовит

2) готовит

3) готовит

22

1) акции

2) акции

3) акции

23

1) пойдет

2) пойдет

3) пойдет

24

1) избранное

2) нравится

3) красиво

25

1) сказал

2) сказал

3) сказал

Задача 2.

Какое слово отличается?

26. A) Молоко B) Помидор C) Стейк D) Стул

27 … A) Блендер B) Телевизор C) Холодильник D) Автобус

28. A) Волк B) Лошадь C) Корова D) Свинья

29. A) апрель B) февраль C) четверг D) сентябрь

30. A) Бег B) Прыжок C) Высокий D) Обучение

31 … A) Завтра B) Вчера C) Сегодня D) Среда

32 … A) Палец B) Носок C) Лоб D) Лапа

33 … A) Водитель B) Библиотекарь C) Ветеринар D) Ленивый

34 … A) Сзади B) Между C) Перед D) Встать

35 … А) Ежик Б) Воробей В) Попугай Г) Лебедь

Пишет

Время 15 минут.

Представьте, что ваш иностранный друг из Интернета попросил вас написать ему о себе. Напишите ему письмо и расскажите о себе.

Не забудьте написать:

    ваш внешний вид

    ваша семья

    ваши друзья

    ваши увлечения

Вы должны написать примерно 50-60 слов.

Лист ответов

F.I ._____________________

Класс ______________________________

Задание 1. Чтение (15 баллов)

1

2

3

4

5

6

7

8

9

10

11

12

13

14

15

Задача 2.Использование английского языка (20 баллов)

16

17

18

19

20

21

22

23

24

25

26

27

28

29

30

31

32

33

34

35

Задание 3: Письмо (10 баллов)

От кого: ____________________________________________________________ Кому: ____________________________________________________________________________________________________________________________________________________________________________________________________________________________________________________________________________________________________________________________________________________________________________________________________________________________________________________________________________________________________________________________________________________________________________________________________________________________________________________________________________________________________________________________________________________________________________________________________________________________________________________________________________________________________________________________________________________________________________________________________________________________________________________________________________________________________________________________________________________________________________________________________________________________________________________________________________________________________________________________________________________________________________________________________________________________________________________________________________ _____________________________________________________________________________________________________________________

Ключи

Чтение

1

т

9

Ф.

2

Ф.

10

т

3

Ф.

11

большой

4

Ф.

12

хвост

5

Ф.

13

парк

6

т

14

мясо

7

т

15

любовь

8

Ф.

Использование английского языка

16

1

26

Д

17

2

27

Д

18

3

28

А

19

3

29

С

20

1

30

С

21

1

31

Д

22

1

32

Д

23

3

33

Д

24

1

34

Д

25

2

35

А

Комплект заданий для учащихся 7-8 классов

Время: 20 минут

Прочтите текст и вопросы ниже. Для каждого вопроса отметьте правильные буквы A, B, C.

Время для кофе

История употребления кофе — одна из величайших и самых захватывающих в истории. Миллионы любителей кофе во всем мире не могут представить себе жизнь без чашки ароматного кофе по утрам. Кофе — это натуральный стимулятор, который заставляет нас чувствовать себя более бодрыми, бдительными и готовыми к концентрации.

Качества мокка, как когда-то назывался кофе, были впервые обнаружены в Эфиопии более тысячи лет назад.Однако не африканцы, а турки и арабы активно поощряли употребление кофе. Привычка пить кофе быстро распространилась по арабскому миру, где кофе завоевал репутацию вина ислама.

Кофе впервые начали выращивать в Йемене. Он был популярен у турок, которые подавали напиток итальянским купцам. В 1615 году торговцы из Венеции привезли кофе в Европу, где он изначально продавался как лекарство. К концу шестнадцатого века кофе пили в крупных европейских городах от Парижа до Лондона. Сейчас во всем мире существуют разные методы приготовления кофе, например, в Турции кофе традиционно варят трижды, а итальянцы являются изобретателями эспрессо и капучино.

1 Обнаружен кофе

a в Европе.

b в Африке.

c в Азии.

2 Популяризация кофе стала

а эфиопы.

б итальянцы.

c турками.

3 Первый выращенный кофе

а в Италии.

б в Турции.

C в Йемене.

4 В Европе кофе был популярен как

лекарство.

б вино ислама.

c натуральным стимулятором.

5 К концу шестнадцатого века люди пили кофе

a в Северной Европе.

b в важнейших городах Европы.

c Только в Париже и Лондоне.

Задача 2

Прочтите праздничную брошюру и решите, является ли каждое утверждение (6-10) верным (A) или ложным (B).

Обведите правильный ответ на листе для ответов.

Ниагарский водопад

Ниагарский водопад расположен между Южным Онтарио (Канада) и штатом Нью-Йорк (США). Первыми жителями, которые поселились в районе Ниагарского водопада, были ирокезы. Они работали фермерами и торговали на берегу реки Ниагара. Когда первые европейцы ступили на эту территорию, они также поселились там. В 19 веке в этом районе произошли самые важные события.Деревня Манчестер (Ниагарский водопад) стала важным центром производственных компаний в первые годы промышленного развития. Туристы начали прибывать в 1820-х годах, и они все еще посещают это спустя почти два столетия.

Самый комфортный способ посетить Ниагарский водопад — это морская прогулка по всемирно известной Деве Тумана. Этот исторический тур на лодке ежегодно принимает миллионы туристов к водопаду с канадской стороны на американскую. Не пропустите две другие важные достопримечательности: прогулку по Белой воде, удивительный тур по глубокому узкому ущелью и Путешествие за водопадом, самостоятельный тур, который доставит вас на смотровую площадку у подножия водопада на лифте. … Затем вы можете отправиться в небольшое путешествие по туннелям, которые ведут за водопад.

После дня захватывающих приключений посетите Ниагарскую площадь со всеми ее магазинами, кинотеатрами и бутиками. Есть много ресторанов, где подают местные продукты и блюда региональной кухни. Ночью посетите местные клубы с живой музыкой и развлечениями. Не забудьте фейерверк над водопадом каждую пятницу и воскресенье!

6. Эта информация предназначена для туристов, которые хотят провести отпуск в Канаде.

7. Увидеть водопад можно только с борта лодки.

8. Первыми, кто поселился здесь, были европейцы.

9. Вы можете попробовать только местные блюда.

10. Вы можете повеселиться ночью.

ИСПОЛЬЗОВАНИЕ АНГЛИЙСКОГО ЯЗЫКА

Время: 10 минут

Прочтите текст ниже и выберите правильное слово для каждого пробела. Для каждого вопроса отметьте

правильная буква — A, B, C или D. Число (0) было сделано для вас в качестве примера.

ОТЦЫ И СЫНЫ

Гарри Реднапп и Фрэнк Лэмпард вместе играли в футбол (0) ……. Футбольный клуб «Вест Хэм» в 1960-е годы. Они (1) …………… лучшие друзья и женились на двух сестрах, поэтому их сыновья, Фрэнк Лэмпард-младший и Джейми Реднапп, (2) ………. …… Отцы мальчиков водили их на футбольное поле после школы и заставляли их (3) ……………. Из-за всей тяжелой работы оба мальчика стал очень хорошим (4) ……………играл в футбол, и оба играли за Англию, как их отцы. Гарри и Фрэнк (5) … Играли в футбол за «Вест Хэм» в 1980-х. Однако из клуба не ушли. Позже Гарри стал менеджером Вест Хэма, а Фрэнк старший стал помощником менеджера. Фрэнк младший (6) …………… их и играл за Вест Хэм (7) …………… Джейми ушел на другую сторону страны и играл за Ливерпуль. Джейми и Фрэнк — друзья, как и их отцы, и (8) ……… .. проводят время вместе, когда могут.Но они не женаты на двух сестрах. Жена Джейми — (9) …………… поп-звезда, у которой нет (10) …………… сестер.

A для

B для

C при

D против

A имел

B сделал

C был

D пошел

A двоюродные братья

B племянники

C братья в законе

9000 9000 D uncle 9000 Поезд B

Обучение C

Поезд D

A на

B до

C с

D на

A продолжение

B остановлено

C началось

D пропущено

A посетило

A посетил

C общий

D присоединился

A так

B но

C потому что

D также

A наслаждайтесь

B хочу

C решите

D согласен

A

B один D это

A несколько

B нет

C несколько

D любое

Перенесите свои ответы на лист ответов!

ПИСЬМО

Время: 20 минут

ИДЕНТИФИКАЦИОННЫЙ НОМЕР

Что вы напишете своему британскому другу о летних каникулах?

В письме напишите о:

  • где вы проводите летний отпуск;

    то, чем вы больше всего любите заниматься;

    Ваши впечатления от отпуска.

Вы должны написать примерно 90-100 слов.

__________________________________________________________________________________________________________________________________________________________________________________________________________________________________________________________________________________________________________________________________________________________________________________________________________________________________________________________________________________________________________________________________________________________________________________________________________________________________________________________________________________________________________________________________________________________________________________________________________________________________________________________________________________________________________________________________________________________________________ ______________________________________________________________________________________________________________________________________________________________________________________________________________________________________________

Идентификационный номер участника

Чтение

Использование английского языка

Комплект заданий для учащихся 9-11 классов

ЧТЕНИЕ

Время: 30 минут

Флоренс Найтингейл

Родители Флоренс Найтингейл были богатыми английскими землевладельцами. Флоренс родилась во Флоренции, Италия, 12 мая 1820 года. Родители назвали ее в честь города, в котором она родилась. Викторианские женщины обычно не ходили в школу, но отец Флоренс считал, что женщины должны получать образование, поэтому обучал Флоренс и ее старшую сестру дома. они изучали итальянский, латинский, греческий, историю и математику.
Когда Флоренс выросла, она заинтересовалась социальными проблемами. когда ей было двенадцать лет, она решила, что хочет сделать что-то полезное в своей жизни. Ей нравилось навещать больных по соседству.
Флоренс была необычной, потому что у большинства викторианских женщин среднего класса не было карьеры, но Флоренс хотела стать медсестрой. Ее родители не хотели, чтобы она стала медсестрой, потому что медсестры обычно были женщинами из рабочего класса. На самом деле они вообще не хотели, чтобы Флоренс работала, они хотели, чтобы она вышла замуж. Однако неодобрение родителей не помешало Флоренс делать то, что она хотела.
Флоренция поехала в Германию, чтобы узнать о сестринском деле. в то время медсестры учились на собственном опыте, а не на обучении.Флоренс ухаживала за больными, выдавала лекарства и помогала во время операций. Она была очень счастлива и объяснила почему: «Мы научились думать о своей работе, а не о себе».
В 1854 году Великобритания вступила в Крымскую войну. Флоренс и бригада из 38 медсестер отправились в Крым, чтобы помочь раненым солдатам. Военные госпитали были грязными и плохо организованными. Флоренция сделала больницы чистыми и безопасными. Количество смертей в больницах снизилось с 40% до 2%. Флоренс была добра и ласкова с солдатами.Когда она гуляла по больнице ночью, она несла лампу, за что и получила прозвище «Дама с лампой».
После войны люди со всего мира спрашивали у нее совета по проектированию больниц. Флоренс считала, что медсестры должны учиться как на опыте, так и на обучении. В 1860 году она открыла свою школу медсестер. Сегодня няни-соловьи по-прежнему заботятся о больных и бедных.

1. Что вы узнали о родителях Флоренс?

A Они были итальянскими гражданами.
B Они дали образование своим детям.
C Они выучили итальянский, латинский, греческий, историю и математику.

2. Чем Флоренс отличалась от большинства других женщин своего времени?

A Она хотела работать.
B Родители не разрешали ей работать.
C Она хотела сделать что-нибудь полезное в своей жизни.

3. Как Флоренс начала свою карьеру?

A Она училась на медсестру в Германии.
B Она работала со своими родителями.
C Она училась сестринскому делу, когда ухаживала за больными.

4. Она дала лекарство в строках 15-16 означает

A Она принесла лекарства нескольким людям.
B Она взяла лекарство из больницы.
C Она делала лекарства для больных.

5. Как Флоренция поменяла больницы?

A Уменьшилось количество больных.
B Она установила более высокие стандарты для больниц.
C Она повсюду ставила лампы в больницах.

Вот некоторая информация о пандах. Прочтите информацию и утверждения ниже и обведите в кружок T, если утверждение верно, F, если утверждение неверно, или U (не указано), если в тексте такой информации нет.

Гигантская панда настолько редка, что сегодня во всем мире насчитывается всего около 1000 особей. Он внесен в список исчезающих видов и в дикой природе встречается только в горах центрального Китая и Восточного Тибета.
Название «панда» относится к любому из двух восточноазиатских животных.Хотя их иногда считают родственниками, гигантская панда обычно классифицируется как медведь, а малая панда, также называемая красной, или обыкновенная панда, относится к семейству енотов.
Гигантская панда имеет длину от 4 до 5 футов (от 1,2 до 1,5 метра) от кончика носа до крупа и короткий хвост. Взрослые люди весят от 165 до 350 фунтов (от 75 до 160 килограммов). У них густая белая шерсть с черным мехом на ногах и ушах. Животные отличаются черными пятнами под глазами и черной полосой на плечах.
Гигантские панды обычно живут одни в бамбуковых лесах. В основном это наземные обитатели, использующие дуплистые деревья или расщелины скал для укрытия, но они также могут лазить по деревьям. Гигантская панда питается в основном побегами и корнями бамбука, цепляясь за тонкие стебли с помощью особой конструкции, напоминающей гробницу, на передних лапах. Иногда они едят мелких животных.
Гигантские панды — популярные животные в зоопарках, но их чрезвычайно сложно разводить. В 1980 году в плену в Китае находилось около 40 человек, а в других странах — около 13 человек.Су-Лин, первая из очень немногих гигантских панд, выставленных на Западе, прибыл в Соединенные Штаты в младенчестве в 1936 году и был популярным аттракционом в зоопарке Брукфилд, недалеко от Чикаго, III, вплоть до своей смерти в 1938 году. В апреле 1972 года китайское правительство отправило двух других, самца и самку, в Национальный зоопарк в Вашингтоне, округ Колумбия,
Малая панда водится в Китае, Мьянме, Сиккиме и Непале в высокогорных лесах и зарослях бамбука. Из-за необычной кошачьей внешности его иногда называют медведем-кошкой, или кошачьим медведем. Длинная мягкая шерсть от рыжевато-красного до темно-каштанового цвета. Лицо и уши в основном белые, но задняя часть ушей красновато-коричневая.
Панды путешествуют парами или небольшими семейными группами. Меньшие панды известны своим в целом мягким нравом. Днем эта панда спит, свернувшись клубочком на дереве с хвостом над головой или сидя на конечности, подложив голову под грудь и между передними лапами. Питается ночью, поедая ростки бамбука, травы, фрукты, другой растительный материал и, иногда, мелких животных.

6) Гигантскую панду надо беречь.

7) Две панды принадлежат к разным семействам.

8) Красные панды живут вместе с енотами.

9) Гигантские панды живут в норах в земле.

10) Вырастить панд в зоопарке непросто.

11) В Чикаго есть особенный аттракцион для панд.

12) Су Линь умер в зоопарке из-за болезни.

13) Панды — ночные животные.

14) Некоторые панды похожи на енотов, другие имеют необычную кошачью внешность.

15) Гигантские панды и малые панды спят по-другому.

ИСПОЛЬЗОВАНИЕ АНГЛИЙСКОГО ЯЗЫКА

Время: 15 минут

Выберите слова, которые лучше всего завершают предложения в тексте

Землетрясения — одни из самых разрушительных ………… .. (1) бедствий. Обычно они ………………… (2) без предупреждения и ……………… (3) в большом …………………… .. (4) жизни и огромном сносе зданий. Кроме того, они могут вызвать разрушительные оползни или создать гигантские приливные волны, которые в …………………….(5) — это колоссальные водные стены, врезающиеся в морские берега с такой силой, что они ………………… (6) разрушают прибрежные города. Однако ………… .. (7) большинство смертельных случаев и серьезных травм ……………………. (8) о том, когда строятся …………………… .. (9).

Чаще всего землетрясение длится от 30 до 60 секунд, поэтому обычно нет времени ………………… (10) смертный ……………………. (11) после начала тряски. Жестокие силы землетрясения вызывают ……………………. (12) сложная цепочка ……………………. . (13) в конструкции здания, когда оно встряхивается, поднимается, толкается или тянется. Высота здания, его форма и строительные материалы являются наиболее важными ………………… (14) при принятии решения о выживании или обрушении конструкции и, как следствие, о жизни или смерти ее ………………… ……….(пятнадцать).

    a) природа b) натуралистическая c) естественная d) естественная

    a) удар b) удар c) падение d) нападение

    a) результат b) эффект c) свинец d) причина

    a) летальный исход b) расточительство c) ущерб d) потеря

    a) факт b) достоверность c) честность d) правда

    a) потенциальный b) возможный c) способный d) возможный

    a) широкий b) широкий c) полный d) обширный

    a) принести b) прийти c) заложить d) сделать

    a) снести b) перемешать c) разрушить d) коллапс

    a) предотвратить b) уклониться c) воздержаться d) сдержать

    a) поддерживать b) поднять c ) результат d) поглощение

    a) увеличение b) включение c) выход d) отключение

    a) активность b) движение c) прогресс d) реакция

    a) факторы b) явления c) точки d) ингредиенты

    a ) поселенцы б) горожане в) жители г) мещане

ПИСЬМО

Время: 30 минут

Прокомментируйте новость:

Возможно, фастфуд — это самый быстрый способ утолить голод. Но по мнению многих специалистов, фастфуд может вызвать серьезные заболевания.

Запись 100-120 слов.

Запомните

Представьте,

Выскажите свое личное мнение о современных экспедициях и аргументируйте свое мнение

,

Сделайте вывод.

Идентификационный номер участника

ЛИСТ ОТВЕТА

Чтение

Использование английского языка

Письмо

__________________________________________________________________________________________________________________________________________________________________________________________________________________________________________________________________________________________________________________________________________________________________________________________________________________________________________________________________________________________________________________________________________________________________________________________________________________________________________________________________________________________________________________________________________________________________________________________________________________________________________________________________________________________________________________________________________________________________________ _______________________________________________________________________________________________________________________________________________________________________________________________________________________________________________________________________________________________________________________________________________________________________________________________________________________________________________________________________________________________________________________________________________________________________________________________________________________________________________________________________________________________________________________________________________________________________________________________________________________________________________________________________________________________________________________________________________________________________________ __________________________________________________________________________________________________________________________________________________________________________________________________________________________________________________________________________________________________________________________________________________________________________________________________________________________________________________________________________________________________________________________________

Ключи к квестам

7-8 классы

Чтение

1b 2c 3c 4a 5b 6F 7F 8F 9T 10T

Использование английского языка

1C 2A 3B 4A 5B 6D 7B 8A 9C 10D

9-11 классы

Чтение

1B2A 3C 4A 5D 6T 7T 8F 9F 10T 11F 12U 13T 14F 15U

Использование английского языка

6. в

11.c

7.d

12.d

8.b

13.d

9.d

14.a

10.a

15.c

Олимпиады по английскому языку становятся все более востребованными во всем мире. Они проводятся как среди учителей, так и среди студентов. Знание английского языка стало преимуществом, открывающим новые горизонты для тех, кто хочет учиться, работать и жить за границей.Кроме того, английский важен во время путешествий и встреч с людьми из разных стран мира.

На данный момент в России проводится довольно много и довольно часто олимпиад по английскому языку. Мероприятия могут проводиться как в школе, так и на региональном или международном уровне. Чтобы добиться высоких результатов на таких мероприятиях, к ним следует тщательно готовиться. Наш международный педагогический портал «Sunshine» рад предложить вам тесты для подготовки к олимпиадам по английскому языку.

Олимпиады по английскому можно пройти онлайн

Педагогический портал «Солнечный свет» приглашает всех желающих принять участие в олимпиадах по английскому языку.Наша цель — раскрыть способность детей и подростков учить и изучать английский язык. Тесты, представленные в этом разделе, помогут многим ученикам и учителям развить свои знания английского языка. Наш портал стремится разрушать препятствия, поэтому онлайн-олимпиаду по английскому языку на 2017-2018 годы можно провести всего пару раз одним щелчком мыши.

В этом разделе вашему вниманию представлено одно из самых эффективных решений для подготовки к олимпиаде. Любой желающий может пройти множество интересных тестов по английскому языку.Такие тесты служат тренажером для подготовки к настоящим олимпиадам. Задания подбираются исходя из действующей школьной программы. Мы представили тесты для 8 класса и по разным темам. Каждый тест состоит из 10 вопросов и нескольких ответов. Вам нужно будет выбрать только один вариант, чтобы проверить правильность ответа, нужно нажать кнопку «Проверить», чтобы сразу понять, правильно вы ответили на тот или иной вопрос или нет.

Олимпиадные задания по английскому языку на портале Sunlight

Наш портал — находка для любого ученика и учителя.Благодаря предоставленным тестам любой желающий сможет подготовиться к олимпиаде, а ответы, представленные в рамках задания, помогут подготовиться к заданию быстрее и лучше. Онлайн-тесты на нашем педагогическом портале можно использовать как тренажер для подготовки к олимпиадам. Отличные и занимательные задания, позволяющие развить мышление. Для всего и про все время неограниченно, то есть не нужно спешить с ответом, здесь можно поразмышлять над задачей и дать верное решение. После теста вы также можете побаловать себя дипломом или свидетельством о прохождении испытаний.

Получите диплом на портале Sunshine

Каждый желающий получить диплом на нашем портале может создать свой кабинет, и там он может создать свой диплом. Низкая стоимость регистрационного взноса поможет вам легко создать свой диплом и скачать его. Таким образом, вы можете не только набраться опыта, выполнив и протестировав себя с помощью онлайн-тестов, но и накопить портфель дипломных сертификатов. Регистрационный взнос совсем невелик, что позволяет любому родителю без проблем оплатить скачивание нескольких дипломов для своего ребенка.

Всероссийская олимпиада школьников

I (школьный) этап

Английский язык

5-6 классы

Комплект учителя

Общее время работы — 60 минут

Максимальное количество баллов -26

В комплект материалов, необходимых для проведения конкурса, входят:

2. Описание процедуры конкурса

3. Текст задания

4. Ключи

5. Лист ответов (для каждого участника)

ЗАДАЧИ ОЛИМПИАДЫ НАЗНАЧЕНЫ НА ТВОРЕНИЕ И НЕ ЯВЛЯЮТСЯ ПРОСТЫМ ПОВТОРЕНИЕМ ФОРМАТА ИСПОЛЬЗОВАНИЯ

Понимание устного текста (аудирование)

У вас есть 10 минут на выполнение заданий, включая ввод ответов на листы ответов.Задание представлено дважды.

Критерии оценки: За каждый правильный ответ дается 1 балл. Задачи 1-4.

Понимание письменного текста (чтение)

Задание по чтению направлено на понимание текста. Студент должен прочитать текст и закончить десять предложений, выбрав правильный ответ из приведенных ниже. На задание дается 20 минут, в которые входит ввод ответов на листы для ответов. Правильность заданий проверяется ключами.

Критерии оценки: За каждый правильный ответ дается 1 балл.Задачи 5-14. Максимальное количество баллов — 10.

Лексико-грамматический тест (Использование английского языка)

Всего в этом блоке 8 вопросов.

Студенты должны продемонстрировать соответствующий уровень владения лексическим материалом и способность использовать его в среде с множественным выбором. Задачи 15-22.

Студент должен показать знание грамматики и лексики и вставить пробелы в предложении из предложенного варианта, найти вариант с грамматической ошибкой, найти букву, подходящую для всех предложенных слов, выбрать правильный перевод предложение, используйте нужный артикль, найдите слово для другой части речи.

За каждый правильный ответ дается 1 балл. Максимальное количество баллов — 8.

Правильность заданий проверяется ключами.

Социокультурная компетентность

Задание состоит из 4 вопросов, направленных на знание учащимися информации регионального и социокультурного характера в рамках школьной программы. Задание оценивается в 1 балл за каждый правильный ответ. Задания 23-26. Максимальное количество баллов — 4. Рекомендуемое время на выполнение задания — 10 минут.

Правильность заданий проверяется ключами.

Порядок проведения конкурса

Общие правила

  • Участники должны сидеть в зале на таком расстоянии друг от друга, чтобы не видеть работы соседей.
  • Участник может взять с собой в зал ручку, стаканы, шоколад, воду.
  • Зрителям не разрешается брать с собой бумагу, справочные материалы (словари, справочники, учебники и др.), Пейджеры и мобильные телефоны, диктофоны, плееры и любые другие технические средства.
  • Во время выполнения задания участник может покинуть зал только в сопровождении сопровождающего.
  • Участник не может покинуть класс с заданием или листом ответов.

Процедура для

Перед тем, как приступить к выполнению заданий, каждому участнику выдается Рабочий лист с заданиями, и на русском языке даются инструкции по порядку проведения олимпиады, заполнению ответов и порядку подачи работы по завершении:

  • Участники заполняют поле «Идентификационный номер участника» на листах с заданиями.
  • СТРОГО ЗАПРЕЩАЕТСЯ УКАЗАТЬ ФАМИЛИ, ЧЕРТЕЖИ ИЛИ ЛЮБЫЕ ЗАЯВЛЕНИЯ НА РАБОТАХ, В противном случае РАБОТА СЧИТАЕТСЯ РАСШИФРОВАННОЙ И НЕ ОЦЕНКА.
  • Исправления не считаются ошибками; однако почерк должен быть разборчивым; Спорные дела (о или о) толкуются не в пользу участника.
  • Задания выполняются только черными или синими чернилами / пастой (красные, зеленые чернила, карандаш запрещены).

Перед тем, как прослушать отрывок, член жюри включает запись и дает участникам возможность прослушать самое начало записи.Затем запись выключают, и член жюри спрашивает публику, все ли хорошо слышат. Если кому-то из участников из зала трудно слышать запись, громкость звука регулируется, и все технические проблемы, влияющие на качество звука, устраняются. Тогда запись начинается с самого начала, не останавливается и слушается до самого конца.

Участники могут просмотреть вопросы перед прослушиванием.

Процедура прослушивания записывается на диск.ПОСЛЕ ПЕРВОГО ПРОСЛУШИВАНИЯ ЧЛЕН ЖЮРИ СНОВА ВКЛЮЧАЕТ ЗАПИСЬ. Две дорожки соответствуют двум заданиям. Запись задания на диск представлена ​​2 раза подряд (допускается техническая пауза 10-15 секунд).

Во время прослушивания участники не могут задавать вопросы членам жюри или покидать аудиторию, так как шум может нарушить процедуру конкурса.

Время конкурса ограничено временем, необходимым для прослушивания присланных заданий.Каждая из задач представлена ​​дважды.

Участникам выдаются тексты заданий и бумага для черновиков. В тексте заданий есть все инструкции по выполнению заданий. Участники получают чистый лист для шашек; проект представлен с заданиями. Однако проверке подлежат только листы задач. Черновики не проверяются.

Член жюри в аудитории должен записать время начала и окончания задания на доске.

За 15 и 5 минут до окончания работы член жюри в зале должен напомнить об оставшемся времени и предупредить о необходимости тщательной проверки работы.

По истечении времени на выполнение заданий собираются листы с заданиями.

Сразу после написания олимпиады работы участников передаются руководителю Школы, который организует проверку работ специально созданной комиссией учебного заведения. Верификация работ проводится под руководством опытного руководителя, который дает инструкции и инструкции группе учителей по верификации. Критерии оценки в виде таблицы раздаются каждому оценщику.Расшифровка работ производится после окончательной оценки работ. По результатам строится рейтинг участников олимпиады.

Критерии оценки

3 место

от 50% до 69%

2 место

от 70% до 90%

1 место

От 91% до 100%

ПРИЛОЖЕНИЕ 1

ШКОЛА ОЛИМПИАДА

5-6 КЛАСС

ПРОСЛУШИВАНИЕ

4 ОЧКА

Задание 1.(Трек 1) Послушайте четырех человек. Для каждого динамика (1–4) выберите его характеристику (A – D).

1. Патрик _____ А. Этот человек умеет играть на гитаре.

2 … Шэрон _____ Б. Этот человек может пользоваться компьютером.

3 … Мэри ______ C. Этот человек любит смотреть теннис по телевизору.

4. Карл _______ Д. Этот человек не любит попсу.

ЧТЕНИЕ

10 ОЧКОВ

Прочтите текст и заполните пропуски, выбрав правильный ответ из предложенных ниже

Фрэнки зеленый и очень высокий.У него желтые глаза, черные волосы и маленький нос. У него также огромные руки и ноги. Он некрасивый. На самом деле, он очень уродлив, и люди называют его монстром. Они его боятся и всегда убегают, когда видят его. Но Фрэнки неплох! Он умный человек и любит читать. У него много книг в маленьком доме.

Он любит писать рассказы. Он продает свои рассказы, чтобы зарабатывать деньги. Ему также нравится пользоваться своим компьютером и Интернет, чтобы писать людям. Они не знают, что он уродлив, и им нравятся его письма.Иногда они просят его фотографию, но Фрэнки никогда ее не отправляет.

Фрэнки увлекается садоводством. В его большом саду много овощей, фруктовых деревьев и роз. Он любит розы! Он разносит в саду хлеб для птиц, так что птиц тоже много. Они его не боятся!

Фрэнки никогда не ест мяса. Ему это не нравится. Он ест овощи, фрукты и хлеб. Поскольку ему не нравится, когда люди убегают от него, он заказывает все свои покупки через Интернет.Магазин доставляет его покупки к нему домой. Таким образом, люди не видят его, и он чувствует себя более счастливым. А когда идет дождь, ему не нужно выходить из дома. Он не любит дождь и боится грома и молний.

5. У Фрэнки….

A. черные волосы и большие руки и ноги

B. зеленые глаза и маленький нос

C. маленький руки и ноги

6. Людям нравится писать Фрэнки, потому что…

A. они не знают, что он уродлив

B.они встречают его в супермаркете

C. они тоже любят мясо

7. Фрэнки….

A. хороший человек

B. злой человек

C. красивый человек

8. Он живет в….

A. большой дом без сада

B. небольшой дом с большим садом

C. небольшой сад

9. Он … много книг.

A. читает

B. говорит

C. делает

10. Фрэнки отправляет свою фотографию….

A. его интернет-друзья

B. никто

C. магазин

11. Птицы….

A. не заходи в сад

B. заходи в сад

C. боимся Фрэнки

12. Фрэнки любит …

A. садоводство

B. есть мясо

C. покупки в торговых центрах

13. Фрэнки идет в…

A. супермаркет

B. театр

C. сад, чтобы положить хлеб для птиц

14.Фрэнки счастлив, когда…

A. люди не видят его

B. он не видит птиц

C. он в супермаркете

ИСПОЛЬЗОВАНИЕ АНГЛИЙСКОГО ЯЗЫКА

8 ОЧКОВ

15. Заполните бланк, выбрав правильный вариант

Часто _______________ зимой у нас дома.

a / snows b / is snowing c / snowed d / snowing e / snow

16. Найдите ошибку в степенях сравнения прилагательных и обведите неправильное число.

a / funny- funnier- самая смешная

b / good- better- the best

c / hot- hoter — самая горячая

d / знаменитая- более известная- самая известная

17.Когда вспомогательный глагол верен?

a / Когда ваша сестра ходит в школу?

б / у не люблю яблоки.

c / Кто ходит в школу каждый день?

D / Они не очень хорошо играют в теннис.

18. Во всех словах отсутствует одна буква. Какая это буква?

Ellow, — oung, — ard, — es.

a / y b / o c / w d / u

19. Выберите правильную пару глаголов для этих предложений.

Вчера утром ______ солнечно.Мы с другом ______ в парк.

a / was, взял b / saw, пошел c / was, играл d / was, пошел

20. Найдите правильный перевод для этого предложения.

Волосы моей матери такой же длины, как волосы моей сестры.

a / Мои волосы длиннее, чем у моей мамы.

б / У моей сестры волосы длиннее, чем у мамы.

c / У моей мамы волосы длиннее, чем у сестры.

d / Волосы моей мамы такие же длинные, как и волосы моей сестры.

21. С этими существительными можно использовать неопределенный артикль «а».

И только в одном случае это будет «ан». Найдите это слово.

a / crocodile b / elephant c / picture d / holiday

22. В этой цепочке слов 4 прилагательных, найдите слово другой части речи.

а / победа б / молодой с / вид д / честный

СОЦИАЛЬНО-КУЛЬТУРНАЯ КОМПЕТЕНЦИЯ

4 ОЧКА

23. Великобритания состоит из

а) Шотландия, Уэльс; б) Англия, Уэльс, Ирландия; в) Англия, Шотландия, Уэльс, Северная Ирландия

24.Чарли Чаплин был…

а. королева б. ученый c. актер d. писатель

25. Хэллоуин празднуется …

а) 1 января б) 31 октября в) 25 декабря

26. Столица Великобритании …

А) Оксфорд б) Ливерпуль в) Лондон

Ключи от квестов.

ПРОСЛУШИВАНИЕ

4 балла

ЧТЕНИЕ

10 баллов

ИСПОЛЬЗОВАНИЕ АНГЛИЙСКОГО

8 баллов

Практика показывает, что зачастую олимпиады становятся терра инкогнито как для учеников, так и для их родителей.И учителя не всегда обладают полной информацией. Цель этой статьи — кратко рассказать вам, что такое олимпиады, как они проводятся и для чего они нужны.

Основные отличия Всероссийской олимпиады от браузеров — это схемы и преимущества.

Всероссийская олимпиада

  • Министерство образования
  • Состоит из четырех ступеней (SHE, ME, RE и ZE)
  • Предоставляет льготы при поступлении
  • Нет необходимости подтверждать преимущества
  • Диплом действителен 4 года

Коричневые олимпиады

  • Принадлежит университетам
  • Состоит из двух этапов (отборочный и финальный)
  • Предоставлять льготы при поступлении
  • Льготы должны быть подтверждены баллами ЕГЭ
  • Дипломы действительны 4 года теории и 1 год практики

Всероссийская олимпиада школьников

Всероссийское — самое престижное событие олимпийского сезона.Обычно они знают об этой олимпиаде больше, чем о других. Причина проста: первый этап общего образования проводится во всех школах , поэтому не слышать о ней практически невозможно. За школьным этапом следуют муниципальный, региональный и выпускной … В школьном этапе может участвовать любой желающий, и для участия в каждом последующем этапе необходимо набрать установленный проходной балл.

Призеры и победители заключительного этапа Всероссийской олимпиады получают различные льготы при поступлении в вузы: БВО (зачисление без экзаменов), 100 баллов по ЕГЭ, баллы за личные достижения. Эти льготы не требуют подтверждения по результатам ЕГЭ.

Призовой фонд и победа на всероссийской гонке — очень сложная задача, часто с непредсказуемым финалом («беспокоился два года, и даже не смог пройти квалификацию в GE»). Поэтому делать ставки только на эту олимпиаду нельзя. Обязательно страхуйтесь с помощью домовых (особенно в 11 классе).

Браун (университетская) Олимпиада

Олимпиады Браунинга, в отличие от Всероссийских олимпиад, не имеют никакого отношения к школе.Их проводят университеты (поэтому одноранговые олимпиады часто называют университетскими). Например, Высшая тестовая олимпиада проводится Высшей школой экономики, Евразийская олимпиада проводится МГЛУ, олимпиада РАНХиГС проводится, как нетрудно догадаться, РАНХиГС. Поскольку школы не занимаются вопросом университетских олимпиад, необходимо самостоятельно отслеживать все связанные с ними мероприятия.

Во-первых, вам нужно знать, какие олимпиады ожидают рассмотрения … Ежегодно Российский совет олимпиад школьников (РСОШ) составляет список олимпиад, который утверждает Минобразования. Этот список и есть список. Если Олимпиада в этом списке, то это список. Если он отсутствует, значит, он не является листингом, и для него не предусмотрены вводные льготы. Олимпиады по английскому языку, включенные в список текущего года, собраны на соответствующей странице сайта Upwego.

Каждой одноранговой олимпиаде присваивается определенный уровень: от I до III.Например, в 2017/18 учебном году было проведено четыре олимпиады по английскому языку I уровня, 7 олимпиад II уровня и 2 олимпиады III уровня (одна из которых проводилась не по всей России). Знание уровня олимпиады необходимо для определения того, какую пользу вы получите.

Набор льгот для обесточенных олимпиад такой же, как и для всероссийских: Британские Виргинские острова, 100 баллов по зачету ЕГЭ или несколько баллов личных достижений. Однако, в отличие от преимуществ Всероссийской олимпиады, любые стимулы к одноранговым олимпиадам должны подтверждаться результатами ЕГЭ : нужно сдать экзамен на 75 баллов и выше (количество баллов может отличаться. в разных вузах, но обычно это ровно 75).

Решив, какие олимпиады входят в список, необходимо зайти на сайты этих олимпиад и изучить правила их проведения. Общая схема для всех одинакова: сначала идет отборочный тур (обычно онлайн), потом идет финал (очный) … Точные даты публикуются на сайтах олимпиад.

Важный момент: для участия в любой одноранговой олимпиаде необходима регистрация, которая на разные олимпиады открывается в разное время.За этим нужно следить, начиная с октября осматривать олимпийские объекты. На некоторых олимпиадах (например, на «Высший тест») регистрация заканчивается до начала квалификационного раунда. Поэтому вопрос регистрации стоит решить заранее, не откладывая до последнего момента. Для регистрации, как правило, требуется определенный набор документов: согласие родителей, справка из школы. Полный список требований указан на сайтах олимпиад. Обязательно изучите их, планируя участие.

Информация о том, когда и как проводится отборочный тур, также публикуется на сайтах олимпиад. Узнав дату и время проведения соревнований, необходимо самостоятельно зайти на сайт Олимпиады и пройти отборочный тур в отведенное для этого время. Набрав проходной балл в квалификационном раунде, вы выходите в финал.

Финалы коричневых олимпиад проводятся очно. У большинства из них есть несколько площадок в разных городах.Необходимо поискать подходящий город для проведения олимпиады, а также поехать туда самостоятельно. Некоторые олимпиады предоставляют иногородним участникам общежития и оплачивают дорожные расходы.

Внимание! Просмотр олимпиад — это личная ответственность участника. Никто не напомнит вам зарегистрироваться, пройти отборочный раунд, посмотреть свои результаты или перейти в финальный раунд. Если забыл, не уследил, не успел — виноваты сами.

Как понять, какие преимущества дает олимпиада?

Решение о том, какие льготы давать олимпиадам, принимают университеты. Таким образом, чтобы узнать, на какие льготы вы имеете право, вам необходимо зайти на сайт интересующего вас университета и найти там документы под названием «Особые права, предоставленные победителям и призерам Всероссийской олимпиады». Всероссийская олимпиада школьников »(при наличии диплома ВОШ) и« Особые права, предоставляемые победителям и призерам олимпиад школьников, включенных в Список олимпиад »(при наличии диплома Коричневой олимпиады).В этих документах подробно описано, какие олимпиадные дипломы на каких факультетах принимаются.

отмечают, что разных факультетов одного вуза устанавливают разные льготы … Например, в 2018 году факультет иностранных языков и межкультурной коммуникации НИУ ВШЭ без вступительных экзаменов принимал только победителей Высшей тестовой олимпиады, а факультет по языку и литературе Ирана в том же университете дал Британские Виргинские острова победителям и призерам олимпиад любого уровня I, а также победителям олимпиад любого уровня II.Также обратите внимание, что опубликованный список льгот может измениться без уведомления в течение года как в сторону расширения, так и в сторону сокращения. Внимательно следите за этим и часто проверяйте.

Срок действия дипломов олимпиады

Дипломы Всероссийской олимпиады действительны 4 года. Это означает, что выиграв Всероссийский в 9 классе, при поступлении в вуз после 11 класса вы сможете воспользоваться всеми преимуществами, которые дает ваш диплом.

Теоретически срок действия дипломов одноранговых олимпиад также составляет 4 года. Однако здесь, в отличие от общероссийских, университеты имеют право указывать класс, в котором был получен диплом, и в большинстве случаев требуют, чтобы он был получен в 11 классе. Таким образом, диплом получен, например, в 9 классе. оценка, формально действительна (не истекли 4 года), но практически бесполезна (нигде не принимается). Точную информацию о том, какие классы учитываются, можно найти на сайтах вузов, в тех же документах об особых правах.

Приходите учиться!

Групповые занятия

К олимпиадам готовлюсь в мини-группах (от 2 до 6 человек). В классе основное внимание уделяется расширению словарного запаса, работе над идиомами, краеведению и другим олимпиадным трюкам. Занятия проходят полностью на английском языке. Домашнее задание — обязательный компонент.

Программы обучения

При подготовке легко упустить из виду что-то важное. Чтобы этого не случилось, я предлагаю вам специальные олимпиадные программы.На протяжении всей программы вы регулярно получаете организованный комплекс упражнений. В отличие от групповых занятий, программы проходят в режиме онлайн, поэтому доступны для любого региона.

Курсы

В отличие от программ, охватывающих сразу несколько направлений, каждый курс посвящен отдельной области. Например, курс «Ассорти олимпиад» даст возможность познакомиться с различными форматами написания олимпиад, а курс «Отчет об олимпиаде» детально раскрывает формат отчета.Полный список доступных курсов можно найти на страницах Курсы, Грамматика, География, Разговорная речь. Также решаем варианты прошлого. Тренироваться нужно каждый день, уделяя подготовке не менее часа. поможет в этом нелегком деле 🙂

Прочтите текст и заполните пропуски 1-6 частями предложений, обозначенными буквами A-G. Одна из частей в списке A-G является избыточной.

Музей дикой природы Линдси

Музей дикой природы Линдси — уникальный центр естествознания и экологического просвещения, где посетители могут послушать крик краснохвостого ястреба, встретиться лицом к лицу с серой лисой и понаблюдать за белоголовым орлом есть ланч.Здесь представлены более пятидесяти видов коренных калифорнийских животных.
Тысячи школьников узнают об окружающей среде в своих классах (1) _________ музея. Для взрослых и детей предлагаются уроки и экскурсии по природе и науке. Более 600 волонтеров помогают кормить и ухаживать за дикими животными, (2) _______. Волонтеры принимают активное участие в работе музея, внося свой вклад (3) ________.
Музей основан местным бизнесменом Александром Линдси.Сэнди, каким его знали друзья, начал учить соседских детей о природе в начале 1950-х годов. Первоначально расположенный в начальной школе, музей начал предлагать детям школьного возраста летние классы (4) ______________.
После почти десятилетней работы музея стало очевидно (5) _____________. С новым домом площадью 5000 квадратных футов музей теперь может развиваться и демонстрировать постоянную коллекцию живых, местных дикой природы и объектов естественной истории.
В музей пришли люди на помощь с дикими животными (6) __________ городской застройки.В ответ на это в 1970 году была начата официальная программа восстановления дикой природы — первая в своем роде в Соединенных Штатах Америки. поездки, ориентированные на мир природы
C. многочасовые услуги по охране дикой природы и сбору средств
D. , которые были ранены или осиротели из-за интенсивных событий
E. , которые требовали общественного внимания и нового здания
F ., а также обучать детей и взрослых природе
G. посредством образовательных программ и выездных экскурсий
Прочтите текст и выполните задания 7-13.

Придайте форму: советы, которым вы можете следовать для более здоровой диеты в кампусе

Я часто бегаю по улицам, и у меня бывают дни, когда я задаюсь вопросом, как найти время, чтобы поесть. Именно тогда ученикам становится легко перейти на фастфуд. Как только вы перестанете жить рядом или у вас возникнут неловкие перерывы между занятиями, фаст-фуд станет доступен почти в каждой столовой на территории кампуса.
Нет сомнений в том, что университет старается сделать более здоровую пищу доступной. В некоторых кафе есть готовые экологически чистые продукты, а Greenhouse позволяет студентам приготовить практически любой вид свежего салата, который они могут придумать. Но я должен задаться вопросом, почему доступно столько фаст-фуда? Многим студентам сложно выбрать дорогой салат из Greenhouse вместо еды из Taco Bell.
Чтобы решить проблему поиска здоровой пищи в университетском городке без ежедневного употребления салата, я делаю три вещи: пользуюсь вариантами питания, которые ранее игнорировал, полностью игнорирую любые заведения быстрого питания и проявляю творческий подход в обеденных залах.
Во-первых, я пользуюсь едой, которую раньше игнорировал в ресторанах кампуса. Вегетарианство и интерес к диете, основанной на принципах справедливой торговли, усложнили составление плана питания во время проживания в университетском городке. При ограниченном выборе еда быстро стареет. Я внимательно посмотрел на меню, чтобы увидеть, какие варианты я упустил. В кампусе я ем овсянку и свежие фрукты вместо смузи. В теплице пробую суп вместо салата. Если у меня есть время утром, я использую дополнительный свайп и собираю обед.Включив это, у меня теперь есть что-то новое, что я могу есть каждый день, и я могу прожить всю учебную неделю, не повторяя обед.
Поскольку я создаю схему закусочных, чтобы поддерживать здоровую диету, не теряя интереса к еде, становится ясно, что я не могу включить в свой план все рестораны на территории кампуса.
Вот где приходит мое второе решение. Я научился полностью игнорировать фастфуд. Если я задержусь перед Panda Express или Sbarro слишком долго, искушение возьмет верх. Я начал игнорировать эти места в середине осеннего квартала, и теперь могу пройти мимо них, не задумываясь.Я знаю, что единственный способ добиться этого — сила воли. Конечно, это никоим образом не означает, что вы никогда не должны позволять себе время от времени посещать эти рестораны. Иногда каждый заслуживает угощения. Я знаю, что никогда не смогу полностью отказаться от жареного картофеля по-животному.
Наконец-то я научился творчески подходить к столовым. Это нелегко сделать за пределами столовой, поэтому это не совсем то, что вам нужно для еды в университетском городке «на ходу». В то же время это может помочь сделать прием пищи менее скучным. Я часто беру вегетарианскую котлету и делаю из нее обертку, а в салат-бар добавляю любую другую начинку, которую хочу.Я использую микроволновую печь, чтобы разогреть в ней все, что хочу. Если вы любитель мяса, вы можете положить кусочки бекона из салатного бара или нарезать жареную куриную грудку и положить в суп или пасту.
Живя в кампусе колледжа, может быть трудно найти способ есть то, что вам нравится и что полезно для вас, при этом пытаясь избежать повторяющейся диеты. Доступно ограниченное количество вариантов, и трудно игнорировать преобладание фаст-фуда. Но если вы попробуете столько мест, сколько сможете, проявите силу воли и включите свое творчество, вы, безусловно, сможете разработать свой собственный план здорового питания, который не будет ощущаться, как будто вы едите одно и то же каждый день.

7. По мнению автора, студенты переходят на фастфуд, когда …

1) у них хорошо выстроен график.
2) им не хватает времени.
3) они живут недалеко от кампуса.
4) у них нет силы воли.

8. Что автор думает о возможностях питания в университетском городке?

1) Она считает, что им нужно давать больше экологически чистых продуктов.
2) Она считает, что они отлично справляются с поставкой салатов по разумной цене.
3) Она не может понять, почему они предлагают так много вариантов быстрого питания.
4) Она считает, что еда из Taco Bell слишком дорогая.

9. Каков авторский метод соблюдения здорового питания?

1) Тщательный выбор ресторана на территории кампуса.
2) Ежедневно меняет ресторан.
3) Ешьте меньше.
4) Пробуем новые блюда по меню.

10. Что НЕ характерно в пищевых привычках автора?

1) Она избегает мяса.
2) Она пропускает обеды.
3) Она внимательно изучает меню.
4) Она пытается разнообразить пищу, которую ест.

11. Каков, по мнению автора, лучший способ перестать есть фастфуд?

1) Посещайте рестораны быстрого питания не чаще одного раза в год.
2) Заменить его жареным картофелем по-звериному.
3) Не ходить возле ресторанов быстрого питания.
4) Придерживаться решения не есть это.

12. Что означает «оно» в абзаце 7 («Точно так же, может помочь …»)?

1) Поиск идеального решения для питания в университетском городке.
2) Самостоятельное приготовление еды.
3) Творчество в столовой.
4) Питание вне столовых.

13. Какова цель статьи?

1) Давать советы по правильному питанию в университетском городке.
2) Убедить читателя в том, что есть одну и ту же пищу каждый день — вредно для здоровья.
3) Классифицировать закусочные на территории кампуса.
4) Доказать, что ежедневное употребление фаст-фуда вредно.

ИСПОЛЬЗОВАНИЕ АНГЛИЙСКОГО

Прочтите текст ниже.При необходимости преобразуйте слова, напечатанные заглавными буквами в конце строк с номерами 14-20, так, чтобы они грамматически соответствовали содержанию текстов.

Эскимо

Не все новое создается мудрыми старыми учеными. Фрэнк Эпперсон изобрел фруктовое мороженое без молока в 1905 году, когда ему было всего 11 лет. Фрэнк (14) _______________ его напиток на крыльце на ночь с палкой для перемешивания в нем.

В ту ночь температура упала и все замерзло, (15) __________ Напиток Фрэнка. Это не помешало ему попробовать это.

18 лет (16) _________________ Фрэнк начал свой бизнес.

Чуть более 1 миллиарда фруктового мороженого (17) ____________ в США каждый год. Из 30 вкусов на выбор, апельсин уже много лет остается фаворитом.

ВЫЙТИ

ВКЛЮЧИТЬ

ПОЗЖЕ

ЕСТЬ

День матери

Прочтите текст ниже.Из слов, напечатанных заглавными буквами в конце строк, обозначенных цифрами 21-26, сформируйте родственные слова так, чтобы они грамматически и лексически соответствовали содержанию текста.

Связь в киберпространстве

В настоящее время большинство из нас ходит на свои компьютеры, чтобы общаться с друзьями, вместо того, чтобы пользоваться телефонами. Отношения в киберпространстве стали нормой для многих людей, даже для большинства (21) _________________ шт.

Удобство и (22) ______________ Интернета просто потрясающе.Если мы хотим посмотреть фильм, найти место, где можно поесть или связаться с другом, мы заходим в Интернет, прежде чем позвонить по телефону.

Критики, однако, говорят, что коммуникация в киберпространстве теряет некоторые важные факторы социальной атмосферы. Мост Интернет (23) ______________ не понимают, что когда мы с кем-то разговариваем, мы получаем от них много сообщений только из-за их тона голоса и языка тела.

Тендер (24) ____________ и зрительный контакт через Интернет невозможен.

Как и все, что мы делаем, без регулярной практики действие может стать незнакомым и (25) _______________.

Есть вероятность, что много может быть (26) ____________ потеряли при общении через Интернет.

ЭФФЕКТИВНЫЙ

ЭКСПРЕСС

РЕАЛЬНЫЙ

Прочтите текст с пробелами, обозначенными цифрами 27-33. Эти номера соответствуют задачам 27-33, в которых представлены возможные ответы.

Диана надеялась уйти к 5:00, чтобы успеть на ферму к обеду. Она пыталась не показывать свои истинные чувства, когда в 16:37 ее заместитель, Фил Хаскингс, представил ей сложный двенадцатистраничный документ, который (27) ______ подпись директора, прежде чем он может быть отправлен клиенту. Хаскинс без колебаний сообщил (28) _______ ей, что они потеряли два аналогичных контракта на той неделе.
Для (29) ___________ по правде говоря, в пятницу всегда было то же самое. В середине дня телефоны замолкали, а затем, когда она думала, что сможет уйти, новый документ падал ей на стол.Диана посмотрела на документ и знала, что до шести часов у нее не будет шанса сбежать.
Диана обожала своих детей. Сначала (30) ________ она выглядела счастливой. Требования быть родителем-одиночкой, а также директором небольшой, но процветающей городской компании означали, что в любой день оставалось (31) _______ минут, чтобы расслабиться. Когда дело доходило до одного из четырех выходных, которые Джеймс и Кэролайн проводили с ее бывшим мужем, Диана пыталась покинуть офис немного раньше, чем обычно, чтобы избежать пробок на выходных.
Она медленно прочитала первую страницу, осознавая, что любая ошибка (32) _____________ в спешке в пятницу вечером может быть (33) _____________ в предстоящие недели. Она взглянула на часы на своем столе, когда подписывала последнюю страницу документа. Он просто показывал 5:51.

27 1) обязано 2) потребовано 3) обязано 4) принуждено
28 1) вспомнить 2) вспомнить 3) вспомнить 4) напомнить
29 1) говорите 2) говорите 3) говорите 4) говорите
30 1) взгляд 2) взгляд 3) взгляд 4) вид
31 1) мало 2) мало 3) много 4) много
32 1) взял 2) сделал 3) провел 4) сделал
33 1) разочарован 2) недоволен 3) извинился 4) пожалел

ПИСЬМО

Вы получили письмо от своей англоговорящей подруги по переписке Джейн, которая пишет:
… В прошлые выходные у моей мамы был юбилей, и у нас был семейный сбор.
Мы развлекали более 25 человек и жили на остатки еды в течение 2 дней после события
. Что вы обычно готовите для особых случаев? Как часто вы
развлекаете членов своей семьи? Вы обычно отмечаете семейные
праздники дома, в кафе или клубе? Почему?
О, пока я не забыл, мой средний брат выиграл наш школьный теннисный турнир …

Напишите письмо Джейн.
В своем письме
ответьте на ее вопросы
задайте 3 вопроса о ее среднем брате

ключей

509 Превышен предел пропускной способности

509 Превышен предел пропускной способности Сервер временно не может обслуживать ваш запрос из-за того, что владелец сайта достиг своего ограничение пропускной способности.

Author: alexxlab

Добавить комментарий

Ваш адрес email не будет опубликован. Обязательные поля помечены *